Top Banner
TEST 17 1 A 29-year-old carpenter comes to the emergency department with his third episode of upper gastrointestinal bleeding in the past year. On both prior occasions, he was found to be bleeding from a duodenal ulcer, and the bleeding was controlled endoscopically. He also reports having ulcers in the distal duodenum, jejunum, and additional ulcers twice in the past 4 years. He does not take any nonsteroidal anti-inflammatory drugs (NSAIDs) and has tested negative for Helicobacter pylori. Which of the following is the most likely cause of his recurrent symptoms? A. Autoimmune destruction of the parietal cells B. Autonomous production of gastrin C. Autonomous production of somatostatin D. Impaired production of the mucous bicarbonate layer in the stomach E. Inhibition of parietal cells secretion Explanation: The correct answer is B. This patient has a history of severe peptic ulcer disease, as manifested by multiple
99
Welcome message from author
This document is posted to help you gain knowledge. Please leave a comment to let me know what you think about it! Share it to your friends and learn new things together.
Transcript
Page 1: TEST 17

TEST 171A 29-year-old carpenter comes to the emergency department with his thirdepisode of upper gastrointestinal bleeding in the past year. On both prioroccasions, he was found to be bleeding from a duodenal ulcer, and the bleedingwas controlled endoscopically. He also reports having ulcers in the distalduodenum, jejunum, and additional ulcers twice in the past 4 years. He doesnot take any nonsteroidal anti-inflammatory drugs (NSAIDs) and has testednegative for Helicobacter pylori. Which of the following is the most likely cause of his recurrentsymptoms?

  A. Autoimmune destruction of the parietalcells

  B. Autonomous production of gastrin

  C. Autonomous production of somatostatin

  D. Impaired production of the mucous bicarbonatelayer in the stomach

  E. Inhibition of parietal cells secretionExplanation:The correct answer is B. Thispatient has a history of severe peptic ulcer disease, as manifested by multipleulcers, ulcers in unusual sites such as jejunum, and ulcers complicated bybleeding. He is not taking nonsteroidal anti-inflammatory drugs (NSAIDs),and he does not have Helicobacter pylori colonizationin the stomach. This suggests that he really does have very high acid secretionby the stomach, which should raise the possibility that he has a gastrin-secretingneuroendocrine tumor that is triggering the high level of acid release. Thetumors are most likely to be in either the pancreas or the proximal smallbowel. The combination of a gastrin-secreting tumor and severe peptic ulcer disease is sometimes called Zollinger-Ellison syndrome, and surgical resectionof the tumor (which may be small and hard to find) usually permits cure ofthe peptic ulcer disease.Autoimmune destruction of the parietal cells (choiceA) causes the disease known as pernicious anemia, which is associatedwith atrophic gastritis.Somatostatin (choice C) inhibits

Page 2: TEST 17

gastrin production and acid secretion, and so excess somatostatin would notbe expected to cause peptic ulcer disease.Impaired production of the mucous bicarbonate layer in the stomach (choice D) is the mechanism whereby NSAIDscause gastric and duodenal ulcers, but this patient is not on these medications.Inhibition of parietal cell secretion (choiceE) would actually reduce ulcer tendencies.2A 2-week post mature baby is born via a vaginal delivery and immediatelyexhibits severe respiratory distress. Previously, green-tinged meconium wasnoted in the amniotic fluid. Which of the following is the most appropriate next step in management?

  A. Emergency tracheostomy

  B. Intubation with mechanical ventilation

  C. Chest x-ray film

  D. Oxygen supplementation by face mask

  E. Suction the mouth and nasopharynxExplanation:The correct answer is E. Meconiumis the fetal stool, which is mostly composed of desquamated cells from thegastrointestinal tract admixed with enough bile to give the soft stool a greenishcolor. A distressed fetus will pass meconium into the amniotic fluid and thenmay aspirate it. The infants often have placental insufficiency as a resultof conditions such as maternal preeclampsia, hypertension, or postmaturity.The aspirated meconium is very irritating to the lungs and causes a chemical pneumonitis. Postmature infants are particularly likely to have severe problems,because the meconium is diluted much less in their small amniotic fluid volume.The most important initial step in therapy is prompt suction of the nasopharynxand mouth to remove the meconium before more (or even any) is aspirated. Thiscan be performed even before the infant is fully delivered, as soon as a headcoated with meconium emerges from the birth canal.Emergency tracheostomy (choice A) isnot warranted at this point.Intubation with mechanical ventilation (choiceB) is deferred until after the nasopharynx and mouth are clearedof meconium.

Page 3: TEST 17

A chest x-ray film (choice C) isnot warranted at this point.Oxygen supplementation by face mask (choiceD) is deferred until after the nasopharynx and mouth are clearedof meconium.3A 28-year-old anesthesiology resident has been having a hard time inhis demanding residency because of the back pain he experiences after longhours of duty. He was involved in a motor vehicle accident 4 years ago andsuffered a serious back injury. One day his wife, who is also a physician, comes home early to find him in bed, wrapped in a blanket complaining of fever,chills, and runny nose. He complains of body aches and soon afterward developsdiarrhea and cramps. The wife confronts him about possible drug use and headmits that he is "finally trying to quit for good." Based on these symptoms, he is most likely experiencing withdrawalfrom which of the following substances?

  A. Alprazolam

  B. Amphetamine

  C. Cocaine

  D. Nicotine

  E. OpioidExplanation:The correct answer is E. Symptomsof opioid withdrawal follow cessation or reduction of prolonged and heavyopioid use and include at least three of the following criteria: dysphoricmood, nausea or vomiting, muscle aches, yawning, diarrhea, lacrimation orrhinorrhea, piloerection, pupillary dilatation, or sweating, fever, and insomnia.The symptoms cause significant distress in social or occupational functioningand are not caused by a general medical condition. Alprazolam withdrawal (choice A)starts after a reduction or cessation of prolonged alprazolam or any sedative,hypnotic, or anxiolytic use. Within two hours, two of the following criteriadevelop: autonomic hyperactivity, increased hand tremor, insomnia, nauseaand vomiting, transient hallucinations, psychomotor agitation, grand mal seizures,or anxiety. The symptoms cause significant distress in everyday functioningand are not caused by a general medical condition.

Page 4: TEST 17

Amphetamine withdrawal (choice B)follows a cessation of two physiologic changes that develop within a few hours:fatigue, unpleasant dreams, insomnia or hypersomnia, increased appetite, psychomotorretardation, or agitation. The symptoms are not caused by a general medicalcondition and cause significant problems in functioning in important areasof life. Cocaine withdrawal (choice C)follows a cessation of heavy and prolonged cocaine use. The symptoms includedysphoric mood and at least two of the following physiologic changes thatdevelop within a few hours or days after cessation: fatigue, increased appetite,psychomotor retardation or agitation, insomnia or hypersomnia, and vivid,unpleasant dreams. The symptoms are not caused by a general medical conditionand cause significant impairment in social or occupational functioning.Nicotine withdrawal (choice D)typically begins within 24 hours after abrupt cessation of nicotine use. Thesigns include at least four of the following: dysphoric mood, insomnia, irritability,difficulty concentrating, anxiety, restlessness, decreased heart rate, andincreased appetite. The symptoms cause significant problems in functioningand are not caused by a general medical condition.4A 27-year-old woman comes to the physician because of fevers and backpain. She states that a few days ago she had burning with urination. Overthe next few days she developed fevers and chills and a pain on the rightside of her back. She has no medical problems and takes no medications. Her temperature is 38.9 C (102 F), blood pressure is 110/70 mm Hg, pulse is 102/minute,and respirations are 16/minute. Examination shows a patient in mild distresswith shaking chills and right costovertebral angle tenderness. Leukocyte countis 18,000/mm3. Urinalysis shows 100 leukocytes/highpowered field. Which of the following is the most appropriate next step in management?

  A. Observation only

  B. Spinal magnetic resonance imaging (MRI)scan

  C. Outpatient management with oral trimethoprim-sulfamethoxazole

  D. Hospital admission and initiation of intravenoustrimethoprim-sulfamethoxazole

  E. Hospital admission and administration of

Page 5: TEST 17

a 2-week course of intravenous tetracyclineExplanation:The correct answer is D. Thispatient has a presentation that is most consistent with pyelonephritis. Patientswith pyelonephritis typically complain of some combination of back pain, fevers,chills, dysuria, nausea, and vomiting. Examination will often show an elevatedtemperature, costovertebral angle tenderness, and an elevated leukocyte count.Urinalysis may demonstrate positive nitrite and leukocyte esterase testing.Urine sediment often reveals white blood cells, red blood cells, and white cell casts. Pyelonephritis can be managed on an outpatient basis if the patientis otherwise healthy, has no complicating factors, and is reliable to returnif her condition worsens. A patient cannot be managed as an outpatient ifthere is any evidence of sepsis. This patient, with her high fevers, shakingchills, and elevated leukocyte count may have sepsis and should thereforebe admitted to the hospital for intravenous antibiotics. Treatment is withIV trimethoprim-sulfamethoxazole, IV ceftriaxone, IV gentamicin with or withoutampicillin, or an IV fluoroquinolone. Once the patient is afebrile, her conditionis improving, and she is able to tolerate oral intake, she may be convertedto an oral antibiotic regimen to complete a 14-day course.Observation only (choice A) wouldnot be correct for this patient. This patient has pyelonephritis, which isunlikely to resolve without antibiotic therapy.Spinal MRI (choice B) is oftenused to evaluate patients with back pain. This patient, however, has backpain that is almost certainly related to a renal infection, therefore spinalMRI would not be necessary.Outpatient management with oral trimethoprim-sulfamethoxazole (choice C) is appropriate in some cases ofuncomplicated pyelonephritis, as explained above. This patient, however, isquite ill and possibly septic. She, therefore, requires hospital admission.Hospital admission and administration of a 2-week course of IV tetracycline (choice E) would not be appropriate. Tetracyclineis not a drug-of-choice in the treatment of pyelonephritis.5A 31-year-old woman smashes her car against a bridge abutment. She sustainsmultiple injuries, including upper and lower extremity fractures. She is fullyawake and alert, and she reports that she was not wearing a seat belt anddistinctly remembers hitting her abdomen against the steering wheel. Her bloodpressure is 135/75 mm Hg, and her pulse is 88/min. Physical examination showsthat she has a rigid and tender abdomen. There is severe tenderness when externalpressure is applied to her abdomen and then suddenly released. She has nobowel sounds. Which of the following would be the most appropriate step in evaluatingpotential intraabdominal injuries?

  A. Continued clinical observation

Page 6: TEST 17

  B. CT scan of the abdomen

  C. Sonogram of the abdomen

  D. Diagnostic peritoneal lavage

  E. Exploratory laparotomyExplanation:The correct answer is E. Thepresence of an "acute abdomen," which this woman has, is an indication forexploratory surgery and prompt repair of the injuries (probably affectinghollow viscera) that have produced the signs of peritoneal irritation.Continued clinical observation (choiceA) would be irresponsible when it is clinically obvious that she already has an acute abdomen. What would one observe for? Development of septicshock? Death?CT scan (choice B) is idealwhen the issue is potential intraabdominal bleeding in a hemodynamically stablepatient who can be safely sent to the radiology department. CT scan might even be a good idea if the picture of acute abdomen were equivocal. But itis not needed here.Diagnostic peritoneal lavage (choiceD) or sonogram done in the emergency department (choice C) are our options when we suspect intraabdominal bleeding andthe patient is too unstable to be sent anywhere. As pointed out above, however,when an acute abdomen has clearly developed, it is time to operate.6An 81-year-old woman with diabetes consults a physician because of severeand persistent earache. Otoscopic examination demonstrates foul-smellingpurulent otorrhea and a red mass lesion of the external ear canal. Biopsyof the mass demonstrates granulation tissue rather than tumor. Which of the following is the most likely causative organism?

  A. Escherichia coli

  B. Haemophilus influenzae

  C. Proteus vulgaris

Page 7: TEST 17

  D. Pseudomonas aeruginosa

  E. Staphylococcus aureusExplanation:The correct answer is D. Externalotitis, or infection of the external ear canal, can be caused by a varietyof organisms, notably including Escherichia coli, Pseudomonasaeruginosa, Proteus vulgaris,and Staphylococcus aureus. There is, however,a severe subtype of external otitis, malignant external otitis, of which youshould be aware. This form is specifically caused by Pseudomonasaeruginosa, and tends to affect elderly diabetics and AIDSpatients, causing the findings illustrated in the question stem. It is particularlyworrisome both because the Pseudomonas organismis so tissue destructive and because it is often highly resistant to mostintravenous antibiotics. (Consult your local microbiology or pharmacologydepartments for advice about local sensitivities if you encounter the condition.)Complications can be devastating, including deafness, facial nerve paralysis,osteomyelitis, and meningitis.Escherichia coli(choiceA) can cause both external otitis and acute otitis media, but doesnot usually cause malignant external otitis.Haemophilus influenzae(choiceB) is an important cause of acute otitis media.Proteus vulgaris(choiceC) can cause external otitis, but does not usually cause the malignantform.Staphylococcus aureus(choiceE) can cause external otitis, but does not usually cause the malignantform.7A 34-year-old man is brought unconscious to the emergency department5 hours after a motor vehicle accident, in which he was ejected out of thecar and hit his head on the pavement. He lost consciousness 3 hours followingthe trauma. On admission, the patient is unresponsive to verbal or painfulstimuli, and his left pupil is fixed and dilated. X-ray films of his headshow a closed left calvarial fracture. CT scan demonstrates evidence of intracranialbleeding. Prior to this accident he was healthy with no significant past medicalhistory. In which of the following compartments is the bleeding most likelyto have developed?

  A. Epidural space

Page 8: TEST 17

  B. Subdural space

  C. Subarachnoid space

  D. Intracerebral

  E. IntraventricularExplanation:The correct answer is A. Theclinical history and imaging findings are consistent with bleeding withinthe epidural space. Epidural hemorrhage is traumatic in origin in most casesand usually associated with fractures of the calvarial wall. This resultsin tearing of one of the epidural arteries (most commonly the middlemeningeal artery), with rapid accumulation of blood betweenthe calvarial bone and the underlying dura. Brief loss of consciousness isoften followed by a lucid period, which may last for a few hours. The patientrelapses into coma because of the enlarging hematoma that displaces the brainand causes uncal herniation. A fixed dilated pupil is the result of the herniateduncus compressing the ipsilateral oculomotor nerve.Hemorrhage within the subdural space (choiceB) is usually due to tearing of the so-called bridging veins. Itis usually of traumatic origin, but cerebral atrophy is an important predisposingcondition. Thus, this form of bleeding develops most commonly in elderly patients,often after minimal trauma.Bleeding in the subarachnoid space (choiceC) manifests with headache of sudden onset (thunderclapheadache) and nuchal rigidity. Rupture of berry aneurysmsin the circle of Willis is the most common cause.Intracerebral hemorrhage (choice D) hasa wide range of etiologies, hypertension being one of the most common. Othercauses include trauma, infections, vascular malformations, bleeding diathesis, neoplasms, and amyloid angiopathy. Clinical manifestations vary, dependingon the location and extent of the bleeding.Intraventricular bleeding (choice E) isusually due to an extension from subarachnoid or intracerebral bleeding andis thus due to the same underlying conditions as the ones mentioned above.8A 4000 g male neonate develops severe cyanosis that begins within minutesof birth. Blood drawn one hour after birth shows metabolic acidosis with respiratoryacidosis. A chest x-ray film shows the aorta positioned directly anteriorto the pulmonary artery, leading to an impression of a narrow mediastinum.

Page 9: TEST 17

An electrocardiogram is normal. Which of the following is the most likely diagnosis?

  A. Aortic valve stenosis

  B. Complete atrioventricular canal defect

  C. Tetralogy of Fallot

  D. Transposition of the great arteries

  E. Underdeveloped (hypoplastic) left ventriclesyndromeExplanation:The correct answer is D. Thisis transposition of the great arteries, in which the aorta arises from theright ventricle and the pulmonary artery arises from the left ventricle. Approximately5% of congenital cardiac anomalies have transposition of the great arteries.Affected babies present within minutes of birth with severe cyanosis and metabolicacidosis secondary to inability to oxygenate tissues. The only exchange ofblood between pulmonic and circulatory systems is typically occurring througha patent ductus arteriosus. The chest x-ray changes illustrated in the questionstem are typical, and are due to superposition of the great vessels (ratherthan the normal side-to-side position). Surgical repair is usually performedwithin 7 to 10 days of life.Aortic valve stenosis (choice A) producesa loud ejection murmur with a prominent systolic click heard best at the upperright sternal border.Complete atrioventricular canal defect (choiceB) can also cause cyanosis at birth, but will show marked ECG changes,sometimes including absent Q waves.Tetralogy of Fallot (choice C) maypresent at birth , with ECG changes showing right ventricular hypertrophyand right axis deviation. Chest x-ray films usually show a small heart witha concave main pulmonary artery.Underdeveloped left ventricle syndrome (choiceE) causes an abrupt onset of severe heart failure with loss ofperipheral pulses at 2-3 days of life.9A 69-year-old woman with a history of diabetes type 2, chronic renalinsufficiency (baseline creatinine of 2.3 mg/dL), and hypertension has shortnessof breath and right-sided pleuritic chest pain. She denies fevers, chills,

Page 10: TEST 17

or cough. Her temperature is 38.1 C (100.6 F), blood pressure is 130/70 mmHg, pulse is 123/min, and respirations are 29/min. Physical examination showsa swollen and tender right calf. Lung examination is normal. Chest x-ray revealsno active disease. At this time, which of the following is the diagnostic test ofchoice for this patient?

  A. D-dimerstudies

  B. Lower extremityvenogram

  C. MR angiogram

  D. Spiral CTscan

  E. Ventilationperfusion scanExplanation:The correct answer is E. Thispatient has a pulmonary embolus (PE). She tells us the classic story of pleuriticchest pain and shortness of breath. Her chest x-ray is clear, as we wouldexpect. Her vital signs reveal tachycardia and tachypnea, which is expected.The test of choice to diagnose PE in most institutions is ventilation perfusionscanning, which looks for mismatches between ventilation and perfusion, whichis what would be found with PE.D-dimer (choice A) is a bloodtest that is very sensitive for diagnosis of PE but not specific. In other words, a negative test can rule out the diagnosis but a positive test doesnot rule in the diagnosis. D-dimer is a marker for blood clotting, so it wouldbe elevated in many conditions.A lower extremity venogram (choice B) is the gold standard for diagnosis of DVT. Since it is a minimallyinvasive procedure with contrast administration, it is rarely needed to helpmake the diagnosis of DVT. Lower-extremity Doppler is a better test for aDVT in this patient; however, the main concern here is for a pulmonary embolism.MR angiogram (choice C) isstill being studied as an option for diagnosis of pulmonary embolus. It isnot yet accepted as an alternative for diagnosis. The usefulness of MR angiogramis limited by the high cost and limited availability of these machines.

Page 11: TEST 17

Spiral CT (choice D) is becomingmore popular for the diagnosis of PE. It is not the appropriate test for thispatient because her elevated creatinine puts her at risk for IV contrast-relatedrenal failure. Spiral CT is also believed to miss some peripheral pulmonaryembolisms, but it is unclear if these peripheral emboli are clinically significant.10A 77-year-old woman comes to the emergency department with 12 hoursof passing bright red blood from her rectum in increasingly large amounts.She becomes dizzy on standing and appears pale. Her blood pressure is 112/60mm Hg while laying supine and 90/56 mm Hg on sitting upright with her legs dangling over the side of the stretcher. Her abdominal examination is normal. Which of the following is the most likely cause of her bleeding?

  A. A bleeding diverticulum in the appendix

  B. A communication between an arteriole andvenule in the cecum

  C. A laceration at the gastroesophageal junction

  D. Perforation of a sigmoid diverticulum

  E. A rectal polypExplanation:The correct answer is B. Acommunication between an arteriole and venule in the cecum is a descriptionof a vascular ectasia, also known as an arteriovenous (AV) malformation. Thisa common cause of painless colonic bleeding in the elderly and may presentwith acute gastrointestinal bleeding (as in this case), chronic gastrointestinalbleeding, or iron-deficiency anemia. These lesions may be difficult to demonstrate,as the bleeding may be intermittent or the colon may be so full of blood thatthe site of origin is obscured. Techniques used to demonstrate bleeding AV malformations include colonoscopy, intraoperative endoscopy, and visceralangiography. Treatment of these lesions is problematic because many patientswill subsequently develop new or recurrent bleeding vessels.A bleeding diverticulum in the appendix (choiceA) does not occur. Diverticular bleeding is very common in theelderly, but it is usually present in the sigmoid colon.A laceration of the gastroesophageal junction (choiceC), i.e., a Mallory-Weiss tear, may produce bright red blood perrectum if the bleeding is profound. However, it more often presents with a

Page 12: TEST 17

typical history of vomiting and retching preceding the bleeding, which isnot described in this question.Perforation of the sigmoid diverticulum (choiceD) does not result in bleeding but will result in acute diverticulitis.Rectal polyps (choice E) rarelyproduce large volume blood loss.11A 34-year-old international investment banker presents with a 3-monthhistory of frequent episodes of loose stool preceded by left lower abdominalcramping. For the past 6 weeks, the stools have become increasingly bloody.On a number of occasions, he has had a sensation of rectal fullness but hasbeen unable to pass any fecal matter. He travels extensively and has beento Asia, India, Pakistan, Germany, and Sweden in past year working on telecommunicationsinfrastructure deals. On physical examination, he has mild tenderness in theleft lower quadrant. A rectal examination reveals grossly bloody stools. Asigmoidoscopy reveals inflammation extending in a symmetric and circumferentialpattern from the anal verge to the distal descending colon. Multiple stooltests are negative for bacterial and parasitic infections. Which of the following is the most likely cause of the patient'ssymptoms?

  A. Crohn disease

  B. Cytomegalovirus

  C. Ischemic colitis

  D. Ulcerative colitis

  E. Yersinia enterocoliticaExplanation:The correct answer is D. Thispatient has the typical subacute or chronic history of bloody diarrhea inassociation with left lower quadrant cramping. He also describes sensationsof tenesmus. Although he has traveled extensively, multiple stool tests arenegative for infectious etiology. Pathologically, ulcerative colitis is characterizedby inflammation and often superficial ulceration that occur without skip lesions,beginning at the anal verge and extending varying distances proximally.Crohn disease (choice A) mayproduce a colitis but is more typically associated with right lower quadrant

Page 13: TEST 17

symptoms and ileitis.Cytomegalovirus (choice B) maycause a picture indistinguishable for ulcerative colitis but is usually seenonly in immunocompromised patients, e.g., those with HIV who have low CD4cell counts.Ischemic colitis (choice C) isusually a segmental colitis and does not usually start at the anal verge. It is more commonly seen in elderly patients or in those with hypercoagulabledisorders.Yersinia enterocolitica(choiceE) may produce diarrhea, infrequently bloody. However, Yersinia favorsinvasion of the terminal ileum and produces the acute onset of right lowerquadrant symptoms.12A 1400-g infant, born at 35 weeks' gestation, is 42 cm in length anda has a head circumference of 28 cm. One day after birth, she becomes veryirritable, tremulous, and inconsolable. Her cry is high-pitched. Her pulseis 174/min. There are no dysmorphic facial features. Her mother had inconsistentprenatal care and has a history of multiple inpatient hospitalizations forsubstance overdoses. To which of the following substances was this newborn most likelyexposed in utero?

  A. Alcohol

  B. Barbiturates

  C. Cocaine

  D. Marijuana

  E. OpiatesExplanation:The correct answer is C. Themost commonly abused drug by pregnant mothers is cocaine. Infants are usuallysmall for gestational age (SGA) and sometimes have microcephaly and neurodevelopmentalabnormalities. Exposed infants are very irritable and inconsolable in the withdrawal period. Their cries are often high-pitched. They are also at increasedrisk of sudden infant death syndrome (SIDS). Periventricular leukomalacia(a CNS ischemic lesion) is also associated with cocaine exposure.Alcohol (choice A) abuse during

Page 14: TEST 17

pregnancy causes fetal alcohol syndrome. It is characterized by failure tothrive, cardiac defects, facial dysmorphic features (such as narrow forehead, microphthalmia, short palpebral fissures, and micrognathia), and neurologicabnormalities.Barbiturate (choice B) useduring pregnancy causes the infant to have limb anomalies, mental retardation,nail hypoplasia, and some dysmorphic features, such as a short nose and alow nasal bridge. It is not usually associated with low birth weight and microcephaly.There is no clear evidence that marijuana (choiceD) use during pregnancy is associated with any teratogenic effectson the infant in humans. In animal studies, however, it has been associatedwith fetal growth restriction and teratogenesis.Opiate (choice E) abuse duringpregnancy is associated with a high incidence of obstetric complications,such as placental abruption, preterm labor, and fetal growth restriction.Tremors, irritability, vomiting, and diarrhea present in the neonatal period.13Three hours after an uneventful appendectomy, a previously healthy (exceptfor appendicitis) 78-year-old man becomes disoriented and confused. He repeatedlyasks the nurses where he is, and his speech pattern is disorganized and rambling.His temperature is 37 C (98.6 F), blood pressure is 120/80 mm Hg, pulse is70/min, and respirations are 18/min. The patient is uncooperative, but shows no physical abnormalities. Mental status examination is not possible becauseof an altered level of consciousness. Laboratory studies show no abnormalities. Which of the following is the most likely diagnosis?

  A. Adjustmentdisorder

  B. Brief psychoticdisorder

  C. Delirium

  D. Delusionaldisorder

  E. DementiaExplanation:The correct answer is C. This

Page 15: TEST 17

patient has delirium, which is also called acute confusional state. It is very common in hospitalized and institutionalized elderly individuals. Itis characterized by a rapid onset of impaired cognition, altered level ofconsciousness, disturbances in attention and psychomotor activity, and alteredsleep-wake cycles. The symptoms tend to fluctuate and it is usually reversiblewhen the underlying disorder is identified and treated. Common causes include psychologic and physical stress (for example, surgery), metabolic disturbances,neoplasms, infections, medications, cerebral and cardiovascular diseases,and withdrawal from alcohol and prescription medications.Adjustment disorder (choice A)is a maladaptive response to a stressful event. The symptoms include mooddisturbances, behavioral changes, and impaired functioning. It usually resolveswithin 6 months.Brief psychotic disorder (choice B)is characterized by the abrupt onset of psychotic symptoms, including hallucinations,delusions, and disorganization with impaired functioning. It is present for more than 1 day, but less than 1 month. It is usually preceded by a stressfullife event.Delusional disorder (choice D)is characterized by the presence of nonbizarre delusions that last for morethan 1 month. Functioning is not usually impaired. Antipsychotic agents andpsychotherapy may be necessary.Dementia (choice E) refersto the slow and insidious onset of cognitive and intellectual deficits with no changes in consciousness. The symptoms are stable and irreversible. Themost common causes are Alzheimer disease and multi-infarct dementia.14A 26-year-old primigravid woman at 35 weeks' gestation comes to thelabor and delivery ward because of painful uterine contractions and a gushof fluid. Sterile speculum examination reveals a pool of clear fluid in thevagina that is nitrazine positive. When the fluid is examined under the microscope,a "ferning" pattern is seen. Cervical examination shows the patient to be4 cm dilated, 100% effaced, and at 0 station. Fetal fingers can be felt alongside the fetal head. External uterine monitoring shows contractions every2 minutes. External fetal monitoring shows the fetal heart rate to be in the130s and reactive. Which of the following is the most appropriate next step in management?

  A. Expectantmanagement

  B. Oxytocinaugmentation

Page 16: TEST 17

  C. Forcepsdelivery

  D. Vacuum delivery

  E. CesareansectionExplanation:The correct answer is A. Thispatient has a compound presentation, which happens when an extremity prolapsesalongside the fetal presenting part. In this case, the compound presentationis the fetal vertex along with a fetal arm. Compound presentation occurs inapproximately 1 in 1000 deliveries and is brought about when the pelvic inletis not completely occluded by the fetal head. Most often this occurs withpremature fetuses. A compound presentation can be allowed to undergo a normallabor and delivery. The prolapsed arm should be left alone, as it will notinterfere with the labor and delivery in most cases. Often the arm will riseout the way as the vertex descends further.Oxytocin augmentation (choice B)would not be appropriate management. This patient is in active labor, withpainful contractions every 2 minutes and 4 cm of cervical dilation. Oxytocinis used in cases in which there is a need to augment labor (e.g., when contractionsare not adequate) or to induce labor (e.g., when there are no contractionspresent.) This patient has adequate contractions.Forceps delivery (choice C)is not indicated at this point. The patient's cervix is not fully dilated,and the presence of the fetal arm, if it persists, would prevent proper applicationof the forceps.Vacuum delivery (choice D)would not be appropriate. As with forceps, vacuum is not used unless the cervixis fully dilated and the vertex is at +2 to +3 station. This patient is only4 cm dilated, and the vertex is at 0 station. There is no fetal or maternalindication at this point for vacuum delivery.Cesarean delivery (choice E)is not indicated. As noted above, most women with a compound presentation,with a hand presenting by the fetal head, can undergo a normal labor and delivery.15 A 60-year-old man has had weight loss and anorexia for the past severalmonths. He also describes vague epigastric discomfort, which is not relievedby antacids. A barium study shows two 2-cm ulcerations with heaped-up edges,on opposite walls of the antrum, described by the radiologist as "kissingulcers." Endoscopy reveals the lesions described, and multiple biopsies return

Page 17: TEST 17

a diagnosis of high-grade lymphoma of the stomach (visceral lymphoma). Before a specific modality of therapy is chosen, it is importantto ascertain which of the following factors?

  A. Depth ofinvasion of the tumor into the gastric wall

  B. Gastricacidity

  C. Presenceor absence of H. pylori infection

  D. Presenceor absence of lymph node metastasis

  E. Presenceor absence of distant metastasisExplanation:The correct answer is A. Gastriclymphoma is highly chemosensitive and radiosensitive, and can be predictablymelted away with either of those therapeutic modalities. Because of the complications of abdominal radiotherapy, chemotherapy is preferred. Before such treatmentis undertaken, it is necessary to ascertain (usually by sophisticated radiologicalimaging modalities) whether the tumor has replaced the entire thickness ofthe gastric wall or not. If it has, one runs the risk of producing gastricperforation when the antineoplastic treatment is applied. In that case, alocal surgical excision should precede the therapy. If the tumor has not replacedthe entire thickness of the gastric wall, surgery is not necessary.Neither gastric acidity (choice B)nor the presence of H. pylori(choice C) changes the need for chemotherapy or radiotherapy forhigh-grade gastric lymphoma. Had the diagnosis been low-grade lymphomatoidtransformation (MALTOMA), eradication of H. pyloriwould have been the appropriate treatment. The presence or absence of lymph node metastasis (choice D) and the presence or absence of distant metastasis (choice E) are usually taken into accountwhen surgical treatment is planned for a cancer. If radiotherapy were considered,that information would also be relevant. But when chemotherapy is used, theeffect is systemic and it would automatically include any metastatic disease.16

Page 18: TEST 17

16A 27-year-old-man without a significant past medical history comes tothe physician for an infertility evaluation. He and his wife have been tryingto conceive for 2 years without success. A complete workup of his wife foundno abnormalities. History reveals that the patient had normal childhood developmentbut late puberty. He had problems throughout his life in school secondaryto what counselors told him was a moderate learning disability. Physical examinationreveals a tall man with long legs and arms that seem out of proportion tothe rest of his body. His testes are small and moderate gynecomastia is present.Early cataracts are noted. Which of the following findings is consistent with this patient'scondition?

  A. Decreasedluteinizing hormone, follicle-stimulating hormone, and testosterone

  B. Elevatedprolactin level

  C. Increasedluteinizing hormone to follicle-stimulating hormone level

  D. Karyotype47,XXY

  E. 21-hydoxylasedeficiencyExplanation:The correct answer is D. Thispatient has Klinefelter syndrome. It is best diagnosed with a 47, XXY karyotype,although variants with 46, XY exist. This condition is characterized by smalltestes, disproportionately long arms and legs (secondary to delayed epiphysealplate closure in puberty), infertility, and gynecomastia. Mental retardationand learning disability are common. Many men are not diagnosed until adulthoodwhen they seek medical attention for infertility. Our patient clearly has these traits.Patients with Klinefelter syndrome often have an unexplained increasein follicle-stimulating hormone and luteinizing hormone. This increase occurseven in patients with normal testosterone levels. Low follicle-stimulatinghormone, luteinizing hormone, and testosterone (choice A) are typical for a secondary form of hypogonadism. Secondaryhypogonadism is not associated with gynecomastia because patients don't havean increase in conversion of testosterone to estradiol, which is stimulated

Page 19: TEST 17

by follicle-stimulating hormone and luteinizing hormone. A prolactinoma (choice B)can certainly explain infertility, although it does not explain the patient's other physical findings. An increased ratio of luteinizing-to-follicle-stimulating hormone (choice C) describes polycystic ovary disorderand is not present in men. Finally, 21-hydroxylase deficiency (choiceE) causes an increase in androgen production, which our patientdoes not have. This condition is associated with late onset female virilization.

17A previously healthy 60-year-old man is referred for neurologic consultationbecause of rapidly progressive memory loss and jerking movements of the upperand lower extremities. Neurologic examination confirms the presence of severecognitive deficits and memory dysfunction. An electroencephalogram shows generalizedperiodic sharp waves. Neuroimaging studies show moderately advanced cerebralatrophy. A cortical biopsy is obtained, which shows diffuse vacuolar changesof the gray matter with reactive astrocytosis but no significant inflammatoryinfiltration. Cerebrospinal fluid analysis is unremarkable. Which of the following is the most likely diagnosis?

  A. Alzheimer dementia

  B. Creutzfeldt-Jakob disease

  C. Glioma

  D. Pseudotumor cerebri

  E. Viral encephalitisExplanation:The correct answer is B. Creutzfeldt-Jakobdisease (CJD) is the most important of the prion-related spongiformencephalopathies. It manifests with a rapidly progressivedementing picture, which is further distinguished by the presence of myoclonicjerking movements and characteristic electroencephalographic changes. Thedisease is rare, with an incidence of 1 case per 1 million in the generalpopulation. Approximately 85% of such cases are sporadic and probably relatedto acquired mutations in the gene coding for prion protein (PrPC).A minority of cases are familial and caused by inherited mutations of the

Page 20: TEST 17

PrPC gene. Rare cases have occurred following implantsof cadaveric tissues (e.g., cornea, dura mater) from infected donors, or intranasal administration of growth hormone of cadaveric origin. A new variant (bovinespongiform encephalopathy) is probably transmitted to humansfrom infected cattle and results in a clinical symptomatology similar to CJD.Vacuolar (spongiform) changes in the gray matter are the defining morphologicchange of CJD.Alzheimer dementia (choice A) hasa protracted course, usually over a period of many years. The clinical pictureis characterized by progressive deterioration of memory functions withoutany specific motor or sensory deficits. A cortical biopsy would reveal senileplaques and neurofibrillary tangles.Glioma (choice C) is the genericdesignation of any primary brain neoplasm of glial origin, including astrocytomasand oligodendrogliomas. A glioma would produce white matter changes detectableon MRI as a space-occupying lesion.Pseudotumor cerebri (choice D) isa rare disorder due to increased intracranial pressure without any apparentcause. This idiopathic condition manifests acutely with headache and othersigns of raised intracranial pressure. It occurs most commonly in obese womenin association with other predisposing conditions, such as renal failure,endocrine diseases, and drug (tetracycline) or vitamin administration.Viral encephalitis (choice E) ischaracterized by perivascular chronic inflammatory infiltration, scatteredmicroglial nodules, neuronal loss, and gliosis.18A 21-year-old professional dancer complains of several episodes of nearloss of consciousness during a performance. She has been in excellent healthand is a principal dancer in the New York City Ballet Corps. She has no familyhistory of coronary artery disease. She does not smoke, and a recent cholesterolprofile was normal. On physical examination, her blood pressure is 142/88mm Hg, and her pulse is 84/min and regular. She has a brisk carotid upstrokewith a double impulse palpable. She has a loud S4 and a harsh systolic murmurheard along the left sternal border. The murmur is accentuated during theValsalva maneuver. An electrocardiogram reveals severe left ventricular hypertrophy. Which of the following is the most appropriate medication in themanagement of this patient?

  A. Captopril

  B. Digoxin

  C. Diltiazem

Page 21: TEST 17

  D. Furosemide

  E. PropranololExplanation:The correct answer is E. Thispatient has the presentation of idiopathic hypertrophic subaortic stenosis,which is a frequent cause of syncope or near syncope in young patients. Shehas a characteristic murmur, which can be distinguished from other systolicmurmurs by its increase with the Valsalva maneuver. Any maneuver that actsto decrease left ventricular size will increase the murmur of idiopathic hypertrophicsubaortic stenosis because the obstructive component increases as the leftventricular cavity shrinks. Beta-blockers, such as propanolol, help relaxthe left ventricular smooth muscle and reduce ventricular outflow obstruction.The angiotensin converting enzyme inhibitor captopril (choiceA), the cardiotropic agent digoxin (choiceB), and the diuretic furosemide (choiceD) are variably effective in patients with dilated cardiomyopathy.However, they are potentially harmful in a patient such as this, who alreadyhas a hypertrophic ventricle and increased ejection fraction.Diltiazem (choice C), a calciumchannel blocker, has lesser effects than propranolol on the relaxation ofventricular smooth muscle.19A 34-year-old woman who is healthy without underlying medical problems comesto clinic with complaints of temperature up to 38.3 C (101 F) and cough withgreenish sputum production for 2 days without any dyspnea. Her pulse is 88/min,and her respiratory rate is 18/min. There is no accessory muscle use or conversationaldyspnea, nor are there wheezes, bronchial breath sounds, rales, or egophonyover the right lower lung fields. Chest x-ray film reveals a right lower lobeconsolidation. Her leukocyte count is 13,000/mm3. Which of the following is the most appropriate pharmacotherapy?

  A. Amoxicillin

  B. Ampicillin-sulbactam

  C. Ceftriaxone

  D. Clarithromycin

Page 22: TEST 17

  E. Clarithromycin plus ceftriaxoneExplanation:The correct answer is D. Thefirst step in the approach to this patient with a community-acquired pneumonia(CAP) is to categorize her condition according to the American Thoracic Society guidelines (2001), which are based on severity of illness, age, comorbidities,and the need for hospitalization. This patient does not meet the criteriafor hospitalization (one of the following is needed: respiratory rate > 30breaths/min, room air PaO2< 60 mm Hg, O2 saturationless than 90% on room air, or bilateral or multiple lobes involved), and sheis younger than 65 years without any comorbidities. The most common organismsare Streptococcus pneumoniae, Mycoplasma pneumoniae, Chlamydiapneumoniae, Haemophilus influenzae, Legionella, and respiratory viruses. Recommended treatment is with a macrolide, such as azithromycin orclarithromycin.Amoxicillin (choice A) doesnot have broad enough coverage to include the organisms listed above.Ceftriaxone (choice C) andampicillin-sulbactam (choice B) areused for hospitalized patients with community-acquired pneumonia.Clarithromycin plus ceftriaxone (choiceE) is typically reserved for patients who are hospitalized with CAP.20A 24-year-old graduate student is found to have acute myelogenous leukemia.Before undergoing a planned bone marrow transplant, he begins aggressive multi-drugchemotherapy. However, his tolerance of this regimen is severely limited byintractable vomiting with each chemotherapy cycle. Which of the following will most likely reduce vomiting in thispatient?

  A. Dopamine

  B. Erythromycin

  C. Omeprazole

  D. Ondansetron

  E. Opiates

Page 23: TEST 17

Explanation:The correct answer is D. Ondansetron,a 5-hydroxytryptamine3 antagonist, is the most potent antiemetic available for chemotherapy-induced vomiting. It is safe and isavailable as an oral or IV medication. Ondansetron has greatly improved theprognosis of many patients who were previously unable to tolerate full-dosechemotherapy because of severe vomiting. It has side effects only infrequently,the most common being constipation.Dopamine (choice A) is actuallya pro-emetic compound. The common mediators of vomiting in the CNS are 5-hydroxytryptamine,dopamine, and acetylcholine.Erythromycin (choice B) alsotends to cause, not relieve, nausea and vomiting. It has motilin-stimulatingproperties in the stomach, leading to increased gastric motility. It is oftherapeutic value in patients with diabetic gastroparesis.Omeprazole (choice C) is ofno value in inhibiting vomiting. It is a proton-pump inhibitor, which is effectivein reducing or eliminating acid secretion.Opiates (choice E) are commonlyused in patients receiving chemotherapy but are actually pro-emetic becausethey stimulate the chemoreceptor trigger zone, which lies within the brainbut outside the blood-brain barrier.21A 61-year-old man is found to have a malignant polyp in the cecum. Heundergoes a right hemicolectomy, and the specimen reveals a 2-cm adenocarcinomaextending into, but not through, the muscularis propria. Eleven lymph nodesare negative, and there is no evidence of distant metastatic spread. He recoversuneventfully and is discharged home on the 6th postoperative day. Four weekslater, he develops sudden onset of abdominal distention with vomiting andthe inability to pass flatus. An abdominal examination reveals distention,diffuse mild tenderness, and hyperactive rushing bowel sounds in the mid-abdomen.A rectal examination reveals no masses and brown, stool that is negative foroccult blood. An abdominal x-ray reveals multiple air-fluid levels in thesmall intestine. Which of the following is the most appropriate next step in management?

  A. Placement of a nasogastric tube

  B. Small bowel series

  C. Barium enema

 

Page 24: TEST 17

D. Abdomen and pelvic CT scan

  E. Surgical explorationExplanation:The correct answer is A. Thispatient has signs and symptoms, as well as x-ray confirmation, of a smallbowel obstruction. Given the histologic findings of the surgical specimenof an early (Duke's B1) lesion, it is highly unlikely that there is recurrenttumor, especially given the prompt occurrence of this bowel obstruction. Themost likely etiology is therefore adhesions, which have caused this obstruction.Placement of a nasogastric tube would be the most appropriate measure to decompressthe bowel. In most of these patients, this approach will lead to gradual andcomplete resolution of the bowel obstruction. Should there be any symptomsof clinical deterioration with the placement of a nasogastric or small intestinaltube, then surgery (choice E) shouldbe considered.Although a small bowel series (choiceB) or a CT scan (choice D) maybe helpful in delineating the site of the adhesion and small bowel obstruction,they would be performed after bowel decompression with a nasogastric tube.Similarly a barium enema (choice C),if thought necessary, would be performed after bowel decompression. However,a barium enema is unlikely to demonstrate an obstructive point since the obstructionis unlikely to be due to anastomotic tumor recurrence, given the early lesionand recent resection.Surgical exploration (choice E) wouldnot be indicated until a trial of conservative therapy with small bowel decompressionand a period of NPO have been attempted.22An infant is brought to the clinic for a routine healthy visit and vaccinations.She is the product of an uncomplicated pregnancy and has been meeting developmentmileposts. She is feeding well, and her mother reports that the baby seemsto be growing well as well. On physical examination, the infant is afebrilewith stable vital signs. She can lift her head to 90 degrees, her eyes followpast the midline, she laughs, regards her own hand and has slight awarenessof her mother. Which of the following is the most likely age of this infant?

  A. 2 months

  B. 4 months

  C. 6 months

Page 25: TEST 17

  D. 12 months

  E. 18 monthsExplanation:The correct answer is B. Theability to lift the head to 90 degrees, eyes crossing the midline, laughingand slight awareness of the caregiver are characteristic childhood developmentlandmarks of a 4 month old infant.A 2 month old infant (choice A) canlift its head to 45 degrees, eyes follow to the midline, vocalizes, smilesand has a state of half-waking consciousness.A 6-month old infant (choice C) canroll over, grasp a rattle, turn to voice, feed self and separate the worldinto a "parent" and "not parent" world.A 12 month old child (choice D) cansit without support, pull to stand, use a pincer grasp, babble, indicate wants,and have stranger anxiety.A 18 month child (choice E) canwalk well, make a tower of 2 blocks, say three words, use a spoon and a cup,have temper tantrums, and bridge gaps by bringing objects to the caregiver.23A 28-year-old pregnant woman with borderline personality disorder isadmitted to the labor and delivery floor because of hypertension and mildedema. During the hospital stay she frequently argues with the day staff,insists on going out to smoke cigarettes, and refuses to obey the hospitalrules. She gets along well with the chief resident and states that he is theonly one who understands her. The staff can no longer handle her attitudeand they complain to the head nurse about the resident being too permissive.The resident feels that the nurses are judgmental and are not doing theirjob. The patient gets more outraged at the nurses, accusing them of beingrigid, cold, and punitive. The attending physician realizes that his teamis obviously divided into 2 groups. Which of the following interventions is the most appropriate andwill most likely be helpful in this situation?

  A. Confrontingthe patient about her manipulative behavior

  B. Dischargingthe patient earlier than planned

Page 26: TEST 17

  C. Educatingthe resident and staff about the nature of splitting

  D. Talkingseparately to the head nurse about "taking it easy" on the resident

  E. Transferringthe patient to a team of nursesExplanation:The correct answer is C. Oneof the typical defense mechanisms that borderline personality patients useis splitting. It is typical that one group or person is idealized and anotherdevalued at the same time. The appropriate intervention is to educate thestaff about it to prevent conflicts and to ensure appropriate patient care.

Confronting the patient (choice A),even though it is used in psychotherapy, is not appropriate in this case andmay increase the conflict.Discharging the patient (choice B)because of splitting is not appropriate. Patient's care should be the firstpriority and should not be jeopardized. The meaning of the behavior and waysto handle it should be explored. Talking separately to the nurse (choiceD) may be perceived by the staff as siding with the patient andthe resident and not really addressing the problem. Transferring the patient to another team of nurses (choice E) is not an adequate solution to the problem, becausethe patient will most likely exhibit the same behavior.24A 32-year-old woman comes to the physician because of amenorrhea. Shehad menarche at age 13 and has had normal periods since then. However, herlast menstrual period was 8 months ago. She also complains of an occasionalmilky nipple discharge. She has no medical problems and takes no medications.She is particularly concerned because she would like to become pregnant assoon as possible. Examination shows a whitish nipple discharge bilaterally,but the rest of the examination is unremarkable. Urine human chorionic gonadotropin(hCG) is negative. Thyroid stimulating hormone (TSH) is normal. Prolactinis elevated. Head MRI scan is unremarkable. Which of the following is the most appropriate pharmacotherapy?

  A. Bromocriptine

Page 27: TEST 17

  B. Dicloxacillin

  C. Magnesiumsulfate

  D. Oral contraceptivepill (OCP)

  E. ThyroxineExplanation:The correct answer is A. Hyperprolactinemia is the cause in approximately10 to 20% of cases of amenorrhea. It is known that elevated prolactin levelsalter the hypothalamic-pituitary-ovarian axis such that ovulation is suppressedand menses do not occur. This patient has amenorrhea, galactorrhea (i.e.,a milky discharge from the breasts), and an elevated prolactin level. Allof these findings are consistent with hyperprolactinemia, likely coming froma pituitary microadenoma. The fact that no mass is seen on the head MRI isalso consistent with a pituitary microadenoma, as small microadenomas maynot be visualized. The treatment of choice for this patient is with bromocriptine. Bromocriptine is a dopamine agonist that has been shown to decrease prolactinlevels and bring about a return of ovulation and menses. The re-establishmentof ovulation is especially important for this patient who wishes to conceive.Dicloxacillin (choice B) isoften used to treat a breast infection, which can occur in a nursing mother. This patient, however, does not have findings consistent with breast infection.Rather, the nipple discharge is secondary to the patient's elevated prolactinlevels.Magnesium sulfate (choice C)is used in obstetrics to prevent seizures in patients with pre-eclampsia andto stop the uterus from contracting in patients with preterm labor. It isnot indicated for the treatment of hyperprolactinemia.The oral contraceptive pill (choice D) would not be appropriate as this is a young woman who wishes tobecome pregnant. If she did not desire pregnancy, the oral contraceptive pillwould be appropriate therapy. One of the major concerns in young women withmicroadenomas is that decreased levels of estrogen will lead to bone lossand the eventual development of osteoporosis. The oral contraceptive pill,by providing daily estrogen and progestin, will help to prevent bone loss.Thyroxine (choice E) is used in patients with hypothyroidism.This patient has a normal TSH and no evidence of hypothyroidism,and would, therefore, not need thyroxine.25

Page 28: TEST 17

25A 71-year-old woman is brought to the physician by her distressed daughter.The daughter relates that, 3 days ago, her mother began to complain of rightupper quadrant abdominal pain. She did not want to eat and "took to her bedsick." The daughter recalls that she complained of chills, nausea, and somevomiting. Physical examination reveals an obtunded, hypotensive, and obviouslyvery sick elderly woman. She has impressive pain to deep palpation in theright upper quadrant, along with muscle guarding and rebound. Her temperatureis 40 C (104 F), and laboratory analysis shows a white cell count of 22,000/mm3 withmultiple immature forms, a bilirubin of 5 mg/dL and alkaline phosphatase of840 U/L. The serum amylase is normal. An emergency sonogram shows multiple stones in the gallbladder, normal thickness of the gallbladder wall withoutpericholecystic fluid, dilated intrahepatic ducts, and common duct with adiameter of 2.1 cm. The sonographer cannot identify stones in the common duct. In addition to intravenous fluids and antibiotics, which of thefollowing is the most appropriate next step in management?

  A. Elective cholecystectomy

  B. Emergency decompression of the commonduct

  C. Emergency cholecystectomy

  D. Emergency surgical exploration of the commonduct

  E. Emergency transhepatic cholecystostomyExplanation:The correct answer is B. Thediagnosis is acute ascending cholangitis. This deadly disease is seen in patientswith long-standing gallstones who get one or more stones in the common duct, where they produce partial obstruction that allows ascending infection. Thefact that the sonographer cannot see the offending stones is irrelevant, asstones in the common duct are often not seen in sonograms. The study showsthe indirect evidence of obstruction: dilated ducts. The chills, very high fever, and extremely elevated alkaline phosphatase are diagnostic. An advancedclinical form is present here, with obtundation, right upper quadrant tenderness,and hypotension. The key component of therapy is immediate decompression ofthe common duct, which is full of pus. How it is achieved is less important.Emergency endoscopic retrograde cholangiopancreatography (ERCP) is usually

Page 29: TEST 17

the first choice, but it can be done by percutaneous transhepatic cholangiography(PTC) or by open surgery.Elective cholecystectomy (choice A) willindeed be needed once the acute problem is resolved. But, if our planningincluded only such elective surgery, the patient would never get it: she wouldbe dead.Emergency cholecystectomy (choice C) wouldnot address the issue of pus in the common duct. She does not have acute cholecystitis,as evidenced by the normal thickness of the gallbladder wall and the absenceof pericholecystic fluid. Furthermore, acute cholecystitis would not haveproduced the impressive levels of alkaline phosphatase.Surgical exploration of the common duct (choiceD) is more than she can tolerate at this time. She does not needall the stones removed with a long operative procedure. She needs the pusout. The rest will come later.Cholecystostomy (choice E) isanother choice for very sick people with acute cholecystitis, which is notthe diagnosis.

26During a camping trip, a 12-year-old boy scout is bitten by a rattlesnakeon his right ankle. Pain and swelling of the bitten area become rapidly intense. While the boy is transported to the closest medical facility, whichof the following is the most appropriate initial step in management?

  A. Apply iceto the bitten part

  B. Apply atourniquet

  C. Give alcoholto the patient

  D. Immobilizethe bitten part in horizontal position

  E. Performincision and suction on the biteExplanation:The correct answer is D. The

Page 30: TEST 17

most crucial measure is to transport the victim of a snakebite to the nearestmedical facility for intravenous administration of antivenin horse serum.The most current recommendations for "field" first aid include immobilizationof the patient and the bitten part in a horizontal position. Systemic spreadof the venom can be retarded if the victim remains as inactive as possible.The bitten limb can be splinted and kept at heart level if feasible. The clinical manifestations depend on the type of venom, whether predominantly cytolytic (rattlesnake and other pit vipers) or neurotoxic (coral snake). Predominantly cytotoxic venom causeslocal pain, redness, swelling, followed by perioral paresthesias and metallictaste; sometimes, shock, and coagulopathy may develop. Neurotoxic envenomation(e.g. coral snake) causes signs and symptoms similar to botulism, i.e. diplopia,dysphagia, ptosis, and respiratory arrest.Any manipulation of the bitten area, such as application of ice (choice A), tourniquet (choice B), or incision and suction (choice E) is strongly discouraged. These measures are either ineffectivein preventing the venom from entering circulation or result in unnecessarylocal trauma and ischemia. Commercially available mechanical devices for localsuction may be helpful if used within 5 minutes, but mouth suction resultsin insignificant venom extraction and may lead to contamination of the woundby oral bacteria.Giving alcohol (choice C)or stimulants to victims of snakebites is also useless or harmful.27A previously healthy, 35-year-old woman comes to medical attention becauseof an unsteady gait. Her temperature is 37 C (98.6 F), blood pressure is 130/70Hg, pulse is 80/min, and respirations are 20/min. Neurologic examination revealsspasticity and decreased vibratory sensation in her right lower extremity,and decreased strength in her left arm. MRI studies show well-demarcated periventricularareas of T2 hyperintensity. A lumbar puncture shows mildly increased protein concentration with oligoclonal IgG bands. Which of the following is the most likely diagnosis?

  A. Cytomegalovirusencephalitis

  B. Multiplesclerosis

  C. Oligodendroglioma

  D. Progressive

Page 31: TEST 17

multifocal leukoencephalopathy

  E. LymphomaExplanation:The correct answer is B. Clinicalsymptomatology, MRI findings, and IgG oligoclonal bands are highly characteristicof multiple sclerosis (MS). It should be emphasized, however, that a diagnosisof definite MS entails identification of multiple episodes of white matterdemyelination separated in space and time. Thus, a single episode of demyelination-related neurologicdeficits is not sufficient to make a diagnosis of MS. However, the characteristicperiventricular distribution of white matter changes on T2-weighted imagesand the CSF changes are typical of MS. The latter is thought to be due toactivation of specific lymphocytic clones against white matter antigens.Cytomegalovirus (CMV) encephalitis (choiceA) affectsseverely immunocompromised patients, especially those with AIDS. CMV has aspecific tropism for ependymal cells and causes periventricular lesions.Oligodendroglioma (choice C)is a primary brain tumor of oligodendroglial origin. It accounts for approximately5% of all primary brain neoplasms. It is usually located in the white matterof the centrum semiovale close to the cortex. Mild lymphocytosis may be seenin the CSF as a nonspecific response known as neighborhoodreaction.Progressive multifocal leukoencephalopathy (choice D) also occurs in immunocompromised patients. It is dueto JC virus (a papovavirus), and is characterized by multiple foci of whitematter destruction scattered throughout the cerebral hemispheres and cerebellum.Lymphoma (choice E) is particularlyfrequent in immunocompromised individuals, such as those with AIDS and thosereceiving immunosuppressant therapy following transplantation. In AIDS patients,brain lymphoma manifests as a space-occupying mass that appears as a ring-enhancing lesion on MRI/CT. In immunocompetent individuals, primary brain lymphoma involvesthe periventricular regions and may be confused with MS. Atypical lymphocytesmay be identified in the CSF.28A 16-year-old boy is brought to the urgent care clinic with a temperatureof 38.4 C (101 F) and low back, wrist, and knee pain. He had a sore throat1 month earlier. His arthritis is diffuse. Pea-sized swellings are noted overthe skin on his knees. He has a serpiginous erythematous area on his anteriortrunk. His blood and throat cultures are negative, and his CBC is unremarkable.His antistreptolysin-O (ASO) titer is high. Which of the following is the most appropriate therapy?

 

Page 32: TEST 17

A. Acetaminophen

  B. Aspirin

  C. Penicillin

  D. Penicillin and aspirin

  E. Supportive careExplanation:The correct answer is D. Thispatient has acute rheumatic fever from group A streptococci. He has migratorypolyarthritis, erythema marginatum, and subcutaneous nodules. Other featuresabsent in this patient are chorea and carditis. His ASO titer indicates recentinfection with Streptococcus. It is advisableto administer penicillin for the infection. The arthritis can be managed withsalicylates.Acetaminophen (choice A) maybe needed to keep his temperature down and prevent a febrile seizure. However,this is not a treatment. Also, a nonsteroidal anti-inflammatory drug is neededfor the arthritis.Aspirin (choice B) will combatthe arthritis, but the infection will remain untreated. Such a course will expose the patient to potential rheumatic heart disease, including severemitral stenosis, in the future.The penicillin alone (choice C) willcover the infection and eliminate the risk of rheumatic heart disease. However,the patient also has a painful arthritis that needs to be addressed.Supportive care (choice E) isnot an option for this patient. Even if he is allergic to penicillin, an alternativecoverage for gram-positive bacteria is mandatory. Prophylactic therapy foran associated movement disorder is not needed.29A 19-year-old college freshman comes to the university health clinicbecause of recurrent episodes of wheezing during basketball practices. Hehas been a starting shooting guard for this team and has usually been hamperedby shortness of breath shortly after beginning practice and during games.The symptoms are accompanied by a nonproductive cough and chest tightness.He denies any symptoms at rest. The symptoms occur whether the practices areindoors or outdoors. On physical examination, he is comfortable and deniesany symptoms. His physical examination is unremarkable. Which of the following cells are most likely to mediate his symptoms?

Page 33: TEST 17

  A. Eosinophils

  B. Lymphocytes

  C. Mast cells

  D. Monocytes

  E. NeutrophilsExplanation:The correct answer is C. Thispatient has symptoms of exercise-induced asthma. Unsurprisingly, he appearsnormal while in the office. The symptoms of exercise-induced asthma are dueto mast cell release of histamines, which degranulate with the initiationof exercise. These symptoms can be prevented with the pre-exercise use ofinhaled cromolyn, which will act to stabilize the mast cells. It is less effectiveonce exercise has begun. Once bronchoconstriction has occurred, symptomatic therapy can be provided with a beta-agonist inhaler.Eosinophils (choice A) areinvolved in allergen-induced asthma.Lymphocytes (choice B), monocytes (choice D), and neutrophils (choiceE) are involved with inflammation, but do not mediate the processof asthma.30A 24-year-old 2nd-year medical student comes to the student health clinicwith complaints of chronic epigastric pain 1 week before taking Step 2 ofthe USMLE. The patient has been awakened from sleep by epigastric pain atapproximately 3 AM on several occasions. Ranitidine has improved his symptomstransiently. Eating generally improves his symptoms for about 30 minutes,but is then followed by increased pain. Which of the following is the most likely cause of his symptoms?

  A. Diffuse gastric mucosal vasoconstriction

  B. Gastrin-producing tumor

  C. Infection or organism invading the antral

Page 34: TEST 17

mucosa

  D. Organism colonizing the gastric antrum

  E. Reduced gastrin releaseExplanation:The correct answer is D. Pepticulcer disease is strongly suggested by this patient's chronic epigastric painthat is severe enough to awaken him at night and is temporarily relieved with ranitidine and eating. In the absence of nonsteroidal anti-inflammatory drug(NSAID) use, the most likely etiology is antral colonization with Helicobacterpylori. This gram-negative organism lives in the mucous layerthat lines the stomach and apparently uses the mucus as its food source. Theresult is focal disruption of the mucosal barrier, predisposing for both chronicgastritis and gastric peptic ulcer formation. In the past, medical personneltended to blame ulcer formation on "stress," with the implication that correctingthe problem involved teaching the patient to be more calm. Now that we morefully understand peptic ulcer disease, we treat the bacterial colonizationas part of our primary therapy.Diffuse gastric mucosal vasoconstriction (choiceA) describes the pathogenesis of stress-induced gastritis.Gastrin-producing tumors (choice B) cancause Zollinger Ellison syndrome, in which the increased circulating gastrinmarkedly increases gastric acid production. These tumors are rare, and patients who have them typically have a history of multiple peptic ulcers, often withcomplications such as perforation or massive bleeding. This patient's historyof symptoms that have worsened during stress is more suggestive of the morecommon form of peptic ulcer disease related to stress and H.pylori colonization.The H. pylori organisms that are associatedwith peptic ulcer disease do not actually invade the antral mucosa (choice C).Increased, but not decreased (choiceE), gastrin release, as described above with respect to Zollinger-Ellisonsyndrome, may cause peptic ulcers.31A 49-year-old man with a long history of alcohol abuse is brought tothe physician by his wife because of gradually increasing confusion. He hadbeen working as an apartment building superintendent until approximately 2weeks ago, when he began feeling drowsy throughout the day and had difficulty sleeping at night. Since then, he has become confused and occasionally disorientedas to time and day. He can recognize his wife and neighbors, but cannot maintaincasual conversations. He denies any fever, chills, or abdominal pain. Sixmonths ago, he was admitted to the hospital with an upper gastrointestinal

Page 35: TEST 17

bleed, which was due to bleeding esophageal varices. His medications include ranitidine, spironolactone, furosemide, and propranolol. On physical examination,he is lethargic and disheveled. His temperature is 36.9 C (98.4 F), bloodpressure is 112/64 mm Hg, pulse is 62/min, and respirations are 18/min. Hehas mildly icteric sclera and bitemporal wasting. His oral mucous membranesare dry. There is no jugulovenous distension. His lungs are clear, and hehas a regular heart rhythm. His abdomen reveals a firm liver edge with a liverspan of 7 cm in the midclavicular line. There is no shifting dullness, anda spleen tip is not palpable. He has no peripheral edema. On a mental statusexamination he recognizes the physician but cannot name the date, the reasonfor his visit, or his home address. Neurologic examination is nonfocal, andasterixis is present. Which of the following laboratory abnormalities will most likelybe found?

  A. BUN of53 mg/dL

  B. Glucoseof 192 mg/dL

  C. Potassiumof 5.7 mEq/L

  D. Serum bicarbonateof 16 mEq/L

  E. Serum calciumof 14.2 mEq/LExplanation:The correct answer is A. Thispatient with alcoholic cirrhosis and portal hypertension has signs and symptomsconsistent with hepatic encephalopathy. The one possible explanation in theabsence of any obvious infection or gastrointestinal bleeding is dehydrationrelated to his diuretic use. Although the diuretics may have been appropriatelyprescribed for ascites, at present he has multiple physical findings suggestingthat he is dehydrated: dry mucous membranes and absence of jugulovenous distention,ascites, and peripheral edema. His pulse does not demonstrate a reflex tachycardia because of the propranolol, which is being used to reduce his portal pressure.In the setting of dehydration, the patient's BUN would be expected to be elevatedand is a frequent exacerbating factor for hepatic encephalopathy.There is no history of diabetes in this patient, so a glucose of 192

Page 36: TEST 17

mg/dL would not be expected (choice B),and this degree of relatively mild hyperglycemia would not explain his mentalstatus.Although the patient is on spironolactone, he is also on furosemide,so it would not be likely that he would be hyperkalemic (choice C).There is nothing to suggest that the patient has a metabolic acidosisso it is unlikely that he would have a low serum bicarbonate (choice D).There is nothing in the history to suggest the possibility of developinghypercalcemia. Furthermore, the patient's furosemide would tend to decrease,not increase, serum calcium (choice E).32A 5-month-old boy is brought to the emergency department by his parentsbecause he has been having crying spells alternating with lethargy for thepast several hours. He was apparently doing fine in the morning, and feedingwell, when he suddenly started crying painfully, drawing his legs to his chestand kicking them in the air. This would last for 10-15 minutes, and then hewould act very tired and drowsy. First they thought it was colic, but afterseveral episodes of similar attacks he started vomiting and passed a stoolthat contained red blood and mucus, and they immediately brought him to thehospital. He has no medical problems and has been developing well. He is breast-fedand the mother had started adding solids over the previous 3 to 4 weeks. Onphysical examination, the patient is a chubby infant in moderate distress,pale, and diaphoretic. His temperature is 36.7 C (98.0 F), pulse is 100/min,and respirations are 22/min. A complete blood count and leukocyte differentialare within normal limits. His abdomen is mildly tender to palpation but notdistended, and there is a sausage-shaped mass in the right hypochondrium.The right lower quadrant feels empty on palpation. His stool is reddish incolor and with hints of gelatinous material. A plain abdominal radiographshows absence of air in the right lower quadrant. Which of the following is the most appropriate next step in thetreatment?

  A. Barium enema

  B. Immediate surgical reduction

  C. Laparoscopy

  D. Manual reduction

  E. Observation in expectation of spontaneous

Page 37: TEST 17

resolutionExplanation:The correct answer is A. Thispatient has intussusception, which occurs when a portion of the gastrointestinaltract slips or telescopes into the portion just distal to it. A barium enemais diagnostic and therapeutic and is the treatment of choice for symptomsof less than 48 hours duration. Most intussusceptions are ileocolic, and mosthave no known cause. Intussusception is most commonly seen in children 5 to24 months of age (range, 3 months to 6 years). Acute onset of cramping, colicky abdominal pain is the hallmark of intussusception. Patients may have vomiting.As the obstruction progresses, the patient may develop fever and lethargy.The classic currant jelly stool (stool with red blood and mucus) is a latefinding. Passing a stool may temporarily relieve pain. A sausage-shaped massmay be palpated in the right upper quadrant on physical examination. If theobstruction becomes complete, there is abdominal distension and shock mayensue. A coil-spring sign is seen as the barium fills the obstruction. Airenemas may also diagnose and treat with greater safety. Ultrasound is helpfulin establishing the diagnosis. Intussusception is an emergency and shouldbe reduced as quickly as possible. Hydrostatic reduction is successful approximately50% of the time for symptoms lasting longer than 48 hours, and 75 to 80% forsymptoms lasting less than 48 hours. It should not be done in the face ofprolonged intussusception, peritonitis, or perforation. Surgery is recommendedin those cases or after failure of hydrostatic reduction. Untreated intussusceptionis almost uniformly fatal. Immediate surgical reduction (choiceB) is not indicated in a patient who has had symptoms for less than 48 hours and has no signs of intestinal necrosis or shock. A hydrostaticreduction (i.e., barium enema) should be attempted first.Laparoscopy (choice C) isemerging as a treatment modality and its use will most certainly grow as familiaritywith the technique increases. It is, however, not yet the first line of treatmentfor intussusception.Manual reduction (choice D) canbe attempted during surgical intervention before resecting the affected bowel.This should be performed only if hydrostatic reduction has failed and thereare no signs of ischemic necrosis of the affected bowel segment. Observationin expectation of spontaneous resolution (choiceE) is not indicated, as untreated intussusception is almost universallyfatal.33A 36-year-old married woman who works in a nursing home complains ofchest pain and pressure. She has had similar episodes in the past, and evaluationswere normal. At the time of the initial workup, she told the physician thatshe has been a "worrier" since childhood and is still quite anxious; the physiciantold her that she has a "nervous heart." She still believes she has seriousheart disease. Since puberty, she suffered from dysmenorrhea, nausea, vomiting,

Page 38: TEST 17

and fatigue, and she "finally" had a hysterectomy with bilateral salpingo-oophorectomy5 years ago because of severe menorrhagia, which was "the last straw." Shesuffers from an "irritable colon" and a "weak stomach." Last year she hadan episode of dizziness and unstable gait that lasted for "some time"; however,a workup done at the time was negative. She suffers from migraine headachesand the standard migraine medications do not help her. She "does not evenbother" going to a regular physician for her back pain, but she sees a chiropractoron a regular basis, which brings some relief. She has been hospitalized seven times in the past 5 years for many of these reasons. Which of the following is the most likely diagnosis?

  A. Conversion disorder

  B. Generalized anxiety disorder

  C. Masked depression

  D. Pain disorder

  E. Somatization disorderExplanation:The correct answer is E. Somatizationdisorder is characterized by the following characteristics: multiple medicalcomplaints resulting in significant diagnostic testing and medical interventions causing impaired social and occupational functioning. The symptoms cannotbe explained by medical findings. The symptoms include: pain in at least foursites, one pseudoneurologic symptom, one symptom of sexual dysfunction, andat least two or more gastrointestinal symptoms. Conversion disorder (choice A) isa constellation of one or more neurologic symptoms associated with psychologicconflict .The symptoms include deficits affecting motor or sensory functionunder voluntary control, disturbances of consciousness, or pseudoseizures.There is a clear temporal association of the onset of symptoms and a stressor.The symptoms are not consciously produced and are not caused by any othermedical or psychiatric condition. Generalized anxiety disorder (choiceB) is defined by unrealistic or excessive worry about activities or life events lasting at least 6 months. In addition, six of the followingmust be present: fatigability, muscle tension, irritability, trouble fallingor staying asleep, difficulties concentrating, or restlessness. The symptomscause significant impairment in everyday functioning.

Page 39: TEST 17

Masked depression (choice C) canbe presented through somatic complaints like cardiovascular, gastrointestinal,urinary, or orthopedic, together with depressive symptoms. The disorder meets criteria for depressive disorder but usually not for somatization disorder.

Pain disorder (choice D) ischaracterized by pain as a prominent feature together with psychologic factorsthat precipitate, exacerbate, and contribute to its severity. It results inimpairment of everyday functioning and is not caused by any other psychiatricor medical condition. If a general medical condition is present, it doesn'thave a role in its onset or maintenance.34A 25-year-old woman comes to the physician for an annual examination.She has been feeling well over the past year. Her past medical and surgicalhistories are unremarkable. Past obstetrical history is significant for theterm vaginal delivery two years ago of a male infant with spina bifida. Examination is within normal limits. The patient states that she would like to try tobecome pregnant within the next few months and wants to know if she needsto start taking any vitamins or medications. Which of the following supplements should this patient take?

  A. Folic acid,4 mg/day starting preconceptionally

  B. Folic acid,4 mg/day starting in the first trimester

  C. VitaminA, 10,000 IU/day starting preconceptionally

  D. VitaminA, 10,000 IU/day starting in the first trimester

  E. No supplementsare neededExplanation:The correct answer is A. Severalstudies have established a relationship between folic acid and the preventionof neural tube defects. The presence of adequate levels of maternal folateappears to play a role in the correct development and closure of the neuraltube. Based on these studies, in 1992, the United States Centers for Disease

Page 40: TEST 17

Control recommended that all women of child-bearing age should consume 0.4mg/day of folic acid starting preconceptionally and continuing for the first3 months of pregnancy. Women who have already had a child with a neural tubedefect, however, fall into a different category. This patient had a childwith spina bifida 2 years ago. For a woman such as this, the recommendationis that 4.0 mg of folic acid be taken daily, starting one month before the planned time of conception and continuing on for the first 3 months of pregnancy.It is believed that this level of supplementation will decrease the risk ofhaving another child with a neural tube defect by 60 to 70%.To recommend folic acid, 4 mg/day starting in the first trimester (choice B) would be incorrect. This patient, because she has had a priorchild with a neural tube defect, should indeed be taking 4 mg/dayduring the first 3 months of pregnancy. However, she shouldn't start whenshe is pregnant, rather, she should be taking this level of folic acid supplementationstarting preconceptionally. It is important that the pregnant woman's folatestores are being supplemented prior to the time of conception.To recommend vitamin A, 10,000 IU/day starting preconceptionally (choice C) or vitamin A, 10,000 IU/day startingin the first trimester (choice D)would be incorrect. First, vitamin A deficiency is very rare in the UnitedStates. Second, vitamin A supplementation with levels of 10,000 IU/day andabove has been associated with birth defects. Supplements taken by pregnantwomen should contain 5,000 IU/day or less.To state that no supplements are needed (choice E) is incorrect. This patient has a previous child witha neural tube defect. She should therefore take 4.0 mg of folic acid/day startingone month before conception and continuing through the first 3 months of pregnancyto help prevent having another child with a neural tube defect.

35A 31-year-old male immigrant from India is found on a routine physicalexamination to have a single, 2-cm nodule in the right lobe of his thyroidgland. The mass is firm, moves up and down with swallowing, and is not tender.The skin of his face and neck is pitted with multiple scars, which suggestsmallpox; however, he explains that the scars are due to very severe acnethat he had as a youngster, for which he eventually received external beamradiation therapy at the age of 14. His thyroid function tests are normal,and a fine needle aspiration (FNA) cytology of the mass is read by the pathologistas "indeterminate." Which of the following is the most appropriate next step in management?

  A. No furthercare is needed

 

Page 41: TEST 17

B. Thyroidfunction tests should be repeated yearly

  C. Thyroidscan and sonogram are needed

  D. FNA shouldbe repeated until it can be read as benign or malignant

  E. ThyroidlobectomyExplanation:The correct answer is E. Thepatient is at high risk for thyroid cancer (young, male, with a single noduleand a history of radiation), and a reading of "indeterminate" in an FNA isa surgical indication.No further care (choice A)is totally wrong. It assumes that normal thyroid function means there is nothingwrong with the thyroid, when in fact thyroid cancer almost never alters thyroidfunction. This choice also assumes that if an FNA is not read as cancer, thepatient does not have that disease.Focusing on function (choice B)as the criterion to do something is wrong for the same reasons.Thyroid scan and sonogram (choice C) were formerly valuable criteria to select surgical candidates (cold solid nodules meant a high risk of cancer), but the FNA provides a higheryield of malignancy in resected specimens, and thus has rendered the othertests obsolete for this purpose.Repeating the FNA (choice D)assumes that, given more cells, the pathologist should be able to distinguishbenign from malignant. The pathologist has no trouble recognizing malignantfeatures in papillary, medullary, or anaplastic cancers of the thyroid, butcannot do so with follicular neoplasms. Follicular adenoma and follicularcarcinoma require a look at the entire specimen to tell them apart.36A 14-year-old boy is evaluated for short stature. He has no significantpast medical history and is considered otherwise healthy by his parents. Heeats a normal diet and has regular meals. His height and weight have beenconsistently at the 5th percentile since early childhood. His physical examinationis normal, with genitalia at Tanner stage 3. Which of the following is the most likely laboratory finding forthis boy?

Page 42: TEST 17

  A. Bone agethat is equivalent to chronologic age

  B. Decreasedcomplement C3 level

  C. Decreasedserum albumin concentration

  D. Decreasedthyroid stimulating hormone

  E. Increasedserum creatinine concentrationExplanation:The correct answer is A. Thisboy most likely has familial short stature (FSS). Children with FSS usuallyhave a normal birth weight and length. At the age of 2-3 years, however, theirgrowth begins to decelerate and drops to around the 5th percentile. The onsetand progression of puberty in children with FSS are normal. Bone age is typicallyconsistent with the chronologic age.A decreased complement C3 level (choiceB) may suggest chronic inflammatory disorders. But, the lack ofany signs or symptoms makes any chronic inflammatory disorder unlikely.A decreased serum albumin concentration (choice C) can be secondary to a variety of conditions, such asnephrotic syndrome and malnutrition. But, the lack of supportive history andphysical examination data makes these conditions unlikely.Decreased thyroid stimulating hormone (choice D) suggests hyperthyroidism as the etiology of the boy'sshort stature, but it is highly unlikely in this case.An increased serum creatinine level (choice E) indicates renal failure, but this is inconsistent withthe child's history and physical examination.37A 75-year-old man is brought to the physician by his daughter becauseof "forgetfulness" and "disorientation" that has been gradually worseningover the past 2 years. She is concerned that he may leave the house and getlost. He has been confused and disoriented from time to time, but last weekon 2 occasions he left the house and the neighbors found him in the street.He is forgetful, misplaces things, and often gets confused when he tries to

Page 43: TEST 17

dress himself. He often insists that he has taken a bath when in fact he hasnot. The family has to take care of his bills because he can no longer managehis money. He has not suffered any major illnesses in the past. A completeevaluation rules out thyroid disease, metabolic and endocrine disorders, depression,drug effects, vitamin deficiencies, vascular disease, infectious disease,and normal pressure hydrocephalus. Mental status examination reveals cognitivedeficits. Which of the following neurotransmitters' activity is most likely deficient in this patient?

  A. Acetylcholine

  B. Dopamine

  C. Glutamate

  D. Norepinephrine

  E. SerotoninExplanation:The correct answer is A. Acetylcholineis known to modulate attention, novelty seeking, and memory by way of basalforebrain projections to cortex and limbic structures. In Alzheimer dementia, there is a deficit of acetylcholine, causing cognitive deficits.Dopamine (choice B) affectsseveral brain functions, mostly by modulating other systems. Dopaminergicprojections from the ventral tegmental area to the cortex play a role in fine-tuningof attention and ability to screen out irrelevant stimuli. Dopamine counteractsacetylcholine and thus plays a role in Parkinson disease and related subcorticaldementias. Glutamate (choice C) has acrucial role in long-term potentiation and thus in formation of long-term memory. Excess stimulation of glutamatergic receptors is seen in stroke, seizures,and neuronal cell death.Norepinephrine (choice D)modulates sleep cycles, mood, appetite, and cognition by targeting thalamus,limbic structures, and cortex. Locus ceruleus is crucial for fine tuning ofthe attentional tone of cortex. In Alzheimer dementia, only if depressionis superimposed, one might want to enhance noradrenergic activity.Serotonin (choice E) is linkedto many functions because of widespread projections and a variety of receptors.It has not been shown to have a prominent role in Alzheimer type dementia,

Page 44: TEST 17

rather in mood disorders that could be associated with dementia.38A 56-year-old smoker without any previous medical history comes to theemergency department complaining of chest pain and shortness of breath withexertion. He is admitted to the hospital for further evaluation. An exercisestress test supports the diagnosis of coronary artery disease and the patientundergoes a heart catheterization. There is diffuse coronary artery diseasebut no clearly stentable lesions. The cardiologist decides that medical managementof this patient's coronary disease is appropriate at this time. The patienthas no allergies and no other medical conditions. His blood pressure (BP)and pulse over 3 days are as follows:Day 1: blood pressure: 146/96mm Hg; pulse 80/minDay 2: blood pressure: 150/90mm Hg; pulse 86/minDay 3: blood pressure: 140/96mm Hg; pulse 73/minWhich is the most appropriateblood pressure medication for this patient?

  A. Diltiazem

  B. Hydrochlorothiazide

  C. Lisinopril

  D. Metoprolol

  E. NifedipineExplanation:The correct answer is D. Thereis clear and convincing evidence that beta-blockers are indicated in the treatmentof patients with coronary artery disease. This is partly because in additionto lowering blood pressure, beta-blockers keep the heart rate slow, whichdecreases strain on the heart and increases myocardial perfusion. Becausethis patient has coronary artery disease and hypertension, the best choicefor this patient is metoprolol. Diltiazem (choice A) is acalcium channel blocker that also has blood pressure lowering properties togetherwith heart rate controlling properties. This class of medications has notbeen shown clearly to decrease mortality in patients with coronary artery

Page 45: TEST 17

disease, so they are not used as first-line agents. They can be consideredin patients who require heart rate control but who cannot tolerate beta-blockers(e.g., asthmatics). Hydrochlorothiazide (choice B)together with beta-blockers can be used as a first-line blood pressure medicationin many patients. It is probably an underused class of medications. Hydrochlorothiazidehas synergistic blood pressure lowering effects with many other classes ofblood pressure medications. Because of our patient's proven coronary arterydisease, a beta-blocker would be a better choice for this patient. Lisinopril (choice C) is inthe class of antihypertensives known as ACE inhibitors. This group of medicationscan be used as a second-line blood pressure agent in the otherwise healthypatient with essential hypertension. ACE inhibitors are indicated as first-lineblood pressure medications in patients with diabetes because they have beenshown to slow the progression of diabetic nephropathy. They also have beenshown to improve survival in patients with known congestive heart failurebecause they act as an after-load reducer. Finally, in postmyocardial infarctionpatients with left-ventricular damage, ACE inhibitors seem to have a beneficialeffect on survival secondary to a remodeling effect on the ventricle.Nifedipine (choice E) isa predominantly peripherally acting antihypertensive. It acts as a peripheral vasodilator and decreases blood pressure. It is known to have the side effectof reflex tachycardia, so it is not first-line therapy for postmyocardialinfarction or angina patients.39A 70-year-old man with an 88 pack-year smokinghistory and hypertension has slowly progressive shortness of breath over thelast 1 to 2 years. Physical examination reveals a cachectic man who is in mild respiratory distress using accessory muscles and "pursed lip" breathing.He is not cyanotic. His breath sounds are decreased without wheezing or rhonchi.His heart sounds are distant. He has no edema. A chest x-ray shows hyperinflationof lung fields with a small heart, flattened diaphragm, and no infiltratesor edema. Pulmonary function testing is most likely to reveal which of thefollowing findings?

  A. Bronchodilatorresponse of approximately 15 to 20%

  B. Decreaseddiffusion capacity of the lung for carbon monoxide

  C. Decreasedresidual volume

Page 46: TEST 17

  D. Decreasedtotal lung capacity

  E. IncreasedFEV1/FVC ratioExplanation:The correct answer is B. Thispatient most likely has COPD, which includes emphysema and chronic bronchitis.Although many patients have features of emphysema and chronic bronchitis,they are separate entities. This patient is described as the classic "pink-puffer"with emphysema. Patients with emphysema-type changes in the lung have irreversiblepermanent dilation of distal airspaces with cartilage destruction. Low diffusioncapacity of the lung for carbon monoxide (DLCO) is expected in emphysema,whereas patients with chronic bronchitis have a normal DLCO. A bronchodilator response (choice A) is the finding of asthma. Chronic obstructive pulmonary diseaseis characterized by irreversible airway obstruction. Patients with emphysema have air-trapping secondary to decreased elasticrecoil in their lungs. This will show increased residual volume (choice C) and total lung capacity (choice D). The ratio of forced expiratory volume in 1 second (FEV1)to forced vital capacity (FVC) is used to measure air movement in and outof the lungs. If this ratio is 80% of expected or less, a diagnosis of obstructivelung disease is suspected. Because our patient has an obstructive lung disease,we would expect FEV1/FVC to decrease, not increase (choice E).40A 53-year-old woman comes to the physician complaining of fatigue overthe past 6 months. During this time, she has also developed pruritus and lost4 pounds. She is not sexually active, and her past medical history is significantonly for Sjögren syndrome. On physical examination, she is afebrile and has mildly icteric sclera. There are excoriations noted on all four extremitiesand trunk and back. The liver edge is smooth and non-tender and measures 9cm at the midclavicular line. There is no ascites, splenomegaly, or peripheraledema. Laboratory results reveal a normal complete blood count, normal electrolytes,and liver function tests with an alkaline phosphatase of 260 U/L (normal,<110 U/L), total bilirubin of 3.1 mg/dL, and normal transaminase levels. Which of the following is the most likely diagnosis?

  A. Acute cholecystitis

  B. Acute hepatitis A infection

Page 47: TEST 17

  C. Bacterial cholangitis

  D. Primary biliary cirrhosis

  E. Primary sclerosing cholangitisExplanation:The correct answer is D. Thiswoman has a classic presentation of primary biliary cirrhosis. It typicallyaffects middle-aged women and will progress gradually to the point of end-stageliver disease over a number of years. The disease is due to an autoimmunedestruction of intrahepatic bile ductules, and the diagnosis is made by liverbiopsy. The serology that should be checked is the antimitochondrial antibody.Primary biliary cirrhosis is often seen in individuals with other autoimmunediseases, such as Sjögren syndrome, pernicious anemia, and Hashimotothyroiditis.Acute cholecystitis (choice A) presentsacutely with right upper quadrant pain and fever and not with chronic fatigueand pruritus.Acute hepatitis A (choice B) maycause jaundice and fatigue, but it is a self-limiting infection and does notlast 6 months.Cholangitis (choice C) isdue to acute obstruction of the common bile duct and presents urgently withfever, right upper quadrant pain, and jaundice (Charcot's triad).Primary sclerosing cholangitis (choiceE) is a sclerosing process of the extra- and intrahepatic ducts,which usually presents in young males with underlying inflammatory bowel disease.41A 51-year-old man undergoes a barium enema as a colon cancer screeningexamination. A 3-mm polyp is found in the ascending colon. It is rounded andsmooth and has met all the radiographic criteria of a benign sessile polyp.No other lesions are seen within the colon. Which of the following is the most appropriate management?

  A. Repeat thebarium enema in 1 year

  B. Schedulea colonoscopy in 1 year

Page 48: TEST 17

  C. Schedulea sigmoidoscopy now

  D. Schedulea colonoscopy now

  E. Schedulea CT scan to rule out nodal involvementExplanation:The correct answer is D. Thefinding of a sessile mass in the colon, even though it is small and has abenign appearance, mandates a colonoscopy for its removal. This would allowa complete excisional biopsy and histologic review. Although frank carcinomais unlikely with a mass this small, either a tubular adenoma or a villousadenoma is conceivably present and should be removed both to establish thediagnosis and to prevent the lesion from eventually progressing to cancer.In addition, the structure of the bowel is sufficiently complex that it isnot uncommon for other small polyps to be present that were not picked upradiologically. These should also be excised, and their histology reviewed.It would be inappropriate to wait 1 year with a known polyp, even witha benign appearance, so follow-up barium enema (choice A) or colonoscopy (choiceB) in 1 year would be inappropriate.A sigmoidoscopy (choice C)would be an inadequate examination to reach the ascending colon to removethis polyp.Extraluminal involvement with such a benign-appearing polyp is highlylikely, so CT scan (choice E) isunnecessary.42A 14-year-old boy is brought to the emergency department because ofsoreness, and weakness in his legs for the past day that has slowly spreadfrom his calves to his thighs. He now complains of weakness in his trunk andarms. On examination he appears tired and lays on the examining table. Histemperature is 37 C (98.6 F), pulse is 48/min, and respirations are 22/min.Both of his legs are diffusely tender. Deep tendon reflexes are absent inthe lower extremities, and sensation is greatly diminished. Which of the following studies is essential for this patient'sdiagnosis?

  A. Creatinine phosphokinase levels

 

Page 49: TEST 17

B. Stool culture for Campylobacterjejuni

  C. Motor nerve conduction test

  D. Cerebrospinal fluid studies

  E. Muscle biopsyExplanation:The correct answer is D. Guillain-Barrésyndrome (GBS) is a postinfectious polyneuropathy that causes demyelinationof BOTH motor and occasionally, sensory nerves. It is classically an ascending paralysis. CSF studies are essential for diagnosis and reveal a protein levelusually twice normal values but with normal amounts of white blood cells,normal glucose level and an absence of pleocytosis (elevated lymphocytes).Autonomic nervous system involvement can produce the bradycardia seen in thisvignette.Creatinine phosphokinase levels (choiceA) may be mildly elevated and sometimes are normal, but are notessential for diagnosis.Stool culture for Campylobacter jejuni (choice B), a well recognized infection associatedwith GBS, is again not essential for diagnosis. By the time the disease presentsstool cultures are often negative.Motor nerve conduction tests (choiceC) would show decreased velocities, but are not specific for GBS.A muscle biopsy (choice E) isnot indicated and can be normal in early stages. Late disease reveals denervationatrophy.43A 32 year-old man with a history of panic disorder who works as a salesmanreturns to his psychiatrist after several weeks of treatment with paroxetine.While he has noted significant improvement in his symptoms, he still notesresidual anxiety when put into social situations in which he has to speakin public or become the center of attention. He has been taking the medicationas prescribed and has not missed any doses. Given that he had some improvement on his current medication regimen, which of the following would be the most appropriate next step in management?

  A. Cognitive-behavior therapy

  B. Electroconvulsive therapy

Page 50: TEST 17

  C. Medication change

  D. Psychoanalysis

  E. Seeing the patient more often for supportivepsychotherapyExplanation:The correct answer is A. Inmany cases of panic disorder, effective treatment involves the use of cognitivebehavior therapy, which incorporates exposing the patient to disturbing stimuliin an attempt to develop coping mechanisms in response to the stimuli.Electroconvulsive therapy (choice B) isnot indicated for use in panic disorder.Medication change (choice C) isnot indicated when the current regimen leads to significant symptom relief,as the amount of time necessary to achieve adequate response on a new medicationdoes not justify a medication switch.Psychoanalysis (choice D) isa long term, time consuming process that is not indicated to treat the acuteresidual anxiety of this patient's panic disorder.Seeing the patient more often for supportive psychotherapy (choice E) would not be as effective a treatmentas cognitive behavioral therapy, according to research on panic disorder.44A 67-year-old woman comes to the physician because of pain with urinationand frequent urination. She has hypertension for which she takes a beta-blocker,but no other medical problems. She states that she is not sexually active.She does not smoke and drinks cranberry juice daily. Examination shows mildsuprapubic tenderness and genital atrophy but is otherwise unremarkable. Urinalysis shows 50 to 100 leukocytes/high powered field (hpf) and 5 to 10 erythrocytes/hpf. Which of the following is the most likely cause of the infection?

  A. Cardiacdisease

  B. Cranberryjuice ingestion

  C. Hypoestrogenism

Page 51: TEST 17

  D. Nephrolithiasis

  E. Sexual intercourseExplanation:The correct answer is C. Thispatient has a presentation that is most consistent with urinary tract infection(UTI). Two of the major risk factors for uncomplicated UTI are sexual intercourseand hypoestrogenism. Sexual intercourse is believed to lead to urinary tractinfection by introducing colonizing bacteria into the bladder. Sexual intercoursehas been shown to increase the number of bacteria in the urine up to ten times.Hypoestrogenism is believed to be a risk factor for UTI because it is knownthat postmenopausal women not receiving estrogen replacement therapy (ERT)are at greater risk for developing a UTI compared with those women who douse ERT. Furthermore, estrogen administration has been shown to prevent recurrentinfection.Cardiac disease (choice A)is a major risk factor for a number of conditions. However, cardiac diseaseis not a known risk factor for UTI.Cranberry juice ingestion (choice B) has, for many years, been believed to help prevent UTIs. Many in the medical establishment viewed this as an "old wives tale." However,there have been many studies that have shown that cranberry juice containssubstances that inhibit bacterial adherence. Moreover, a recent study showedthat elderly women that drank cranberry juice have lower rates of pyuria andbacteriuria and a decreased need for antibiotics.Nephrolithiasis (choice D)can be a risk factor for the development of an eventual infection, but itis not as common a risk factor as is hypoestrogenism or sexual intercourse.Furthermore, this patient has no evidence of nephrolithiasis, which typicallycauses severe to excruciating episodes of pain.Sexual intercourse (choice E),as noted above, is a well-known risk factor for the development of a UTI.Sexually active women with recurrent UTIs may be treated with a single doseof antibiotic prophylactically after intercourse. This patient, however, hasstated that she is not sexually active.45A 61-year-old man with a known history of hemorrhoids calls his physiciancomplaining of some anal itching and discomfort. The symptoms have been intermittentfor the past 2 months but have recently gotten worse. His pain is mostly towardthe end of the day and at times is so uncomfortable he finds himself sittingleaning to his side so that he does not make the situation worse by sittingon his rear. Bowel movements have become difficult secondary to the pain andthere is now a problem with constipation. The patient had a routine healthmaintenance examination 3 months ago and his next appointment is 9 months

Page 52: TEST 17

away. The physician is very busy in the office that day and only has a quick moment to speak with him. Which of the following is the most appropriate next step in management?

  A. Have patientcome in for examination

  B. Prescribeanalgesics for symptomatic relief

  C. Prescribesitz baths and stool softeners over the phone

  D. Providereassurance and have patient keep appointment in 9 months

  E. Scheduleexcision of thrombosed hemorrhoid under anesthesiaExplanation:The correct answer is A. Thisquestion evaluates many important aspects. First, never attempt to diagnosea patient without a complete history and physical examination. This goes fora patient calling over the phone or one sitting in front of you at the office.Also, it is inappropriate to provide pain medication to someone whom you havenot examined (choice B). The analgesicsmay easily control the pain but also prevent someone from seeking the appropriatetreatment for the underlying problem that may be much more serious.Hemorrhoids are classified as either internal or external, based ontheir location. They occur when the veins of either the internal or externalhemorrhoid plexus become enlarged and the supporting muscles deteriorate.Both types are very common and are associated with increased hydrostatic pressurein the portal venous system, such as during pregnancy, straining at stool,or cirrhosis. The most common manifestation of hemorrhoids is painless, brightred rectal bleeding that occurs with bowel movement. Some complain of a feelingof incomplete evacuation. Definitive diagnosis is made by physical examination.If possible, the patient is asked to strain as if defecating and the anus is examined for prolapsing veins. Anoscopy and proctoscopy may be performedto rule out coexisting pathology. Although this patient has a known historyof hemorrhoids and his complaints are consistent with a "flare-up" of hishemorrhoids, you as the physician have an obligation to determine that thereis nothing more serious going on with this patient. The patient did not offer

Page 53: TEST 17

these symptoms at his last physical, and waiting 9 months to examine him againis much too long (choice D). Therefore,it is necessary for him to come to the office for a more thorough historyand physical examination. Because it will be necessary to rule out cancer,this patient will need a digital rectal examination, anoscopy, and flexiblesigmoidoscopy. Once malignancy has been ruled out, conservative treatmentwith sitz baths, stool softeners, and stool bulking agents may be used (choice C).Thrombosis is one of the complications of hemorrhoids. This occurs whenthe hemorrhoid has prolapsed and the normal blood within the vein has becomeacutely thrombosed. This leads to severe pain. The treatment for an acutelythrombosed hemorrhoid is incision and extraction of the clot, followed bycompression of the incised area (choice E). This patient's pain does not seem this severe; however, do notschedule surgery without a good history and physical examination.46A 64-year-old man is brought to the emergency department by his wifebecause of severe epigastric distress and nausea that began soon after dinner.The patient says that it is probably "just indigestion," but he appears pale,sweaty, and restless. He has no significant past medical history and he doesnot take any medications. His blood pressure is 130/70 mm Hg, pulse is 110/minwith occasional premature beats, and respirations are 20/min. Physical examinationreveals lungs that are clear to auscultation and heart sounds that are slightlysofter than normal, but there are no murmurs. Which of the following is the most appropriate initial diagnosticstep?

  A. Chest x-ray

  B. Complete blood count

  C. Echocardiography

  D. Electrocardiography

  E. Upper endoscopyExplanation:The correct answer is D. Notinfrequently, the symptoms of myocardial ischemia are interpreted by patientsas of gastrointestinal origin (gastric upset, heartburn, indigestion). However,myocardial infarction (MI) is usually accompanied by objective signs of sympatheticactivation, including sweating, anxiety, tachypnea, and tachycardia. Additional

Page 54: TEST 17

symptoms may include light-headedness, dyspnea, orthopnea, cough, nausea,and syncope. An electrocardiogram is imperative in any patient presentingwith this symptomatology. A normal tracing is rare with acute MI.Chest x-ray (choice A) wouldnot be useful in this case, since there are no physical signs pointing to the lungs as a possible source of symptoms.A complete blood count (choice B) wouldlikely show leukocytosis on the second day following MI, but it is of littlediagnostic value in the initial approach. On the other hand, serum CK-MB isoenzyme levels are elevated within 6 hours after the onset of symptoms.Echocardiography (choice C) isa helpful adjunctive tool in demonstrating abnormal motility of ischemic segmentsof the ventricular wall following an MI. However, it is not used as an initial diagnostic test prior to obtaining an ECG.Upper endoscopy (choice E) withbiopsy is the standard diagnostic tool in the study of gastroesophageal refluxdisease, which manifests with postprandial burning epigastric or substernal pain relieved by antacids.47An 8-month-old boy is diagnosed with obstructive hydrocephalus. He wasborn prematurely after a 26-week gestation to a 25-year-old primigravida anddeveloped seizures and hypoxemia soon after birth. Which of the following is the most likely underlying cause of thisclinical course?

  A. Arnold-Chiarimalformation

  B. Germinalmatrix hemorrhage

  C. Kernicterus

  D. Periventricularleukomalacia

  E. UlegyriaExplanation:The correct answer is B. Theclinical manifestations are compatible with germinal matrix hemorrhage (GMH).The germinal matrix is a highly vascularized layer of neuroectodermal precursors

Page 55: TEST 17

lining the ventricular cavities, which is most developed between 22 and 30weeks of intrauterine life. The vessels of the germinal matrix are especiallyvulnerable to hypoxic insults. Babies born prematurely are thus at high riskof hemorrhage in the germinal matrix region, as they often suffer from pooroxygenation. The pathophysiologic consequences depend on the severity of GMH.Severe forms of GMH extend into the ventricles and result in death, or obstructive hydrocephalus and neurologic deficits later in life.Both types of Arnold-Chiari malformation (choice A) (more frequently type 2) may lead to obstructive hydrocephalusdue to compression of the fourth ventricles by the downward displacement ofthe cerebellar tonsils.Kernicterus (choice C) refersto brain damage due to accumulation of unconjugated bilirubin in the centralnervous system. This complication occurs in babies suffering from severe formsof jaundice, e.g., those with erythroblastosis fetalis.Periventricular leukomalacia (choiceD) results from ischemic damage to the periventricular white matterin premature babies. In premature infants, the periventricular white matteris more vulnerable to hypoxic injury.Ulegyria (choice E) is a complicationrelated to intrauterine hypoxic injury. The depth of the sulci is more proneto ischemia than the crests of the gyri during intrauterine life. A fetussuffering from early cerebral ischemic damage will thus develop atrophy ofthe base of the gyri, which then acquire a mushroom-like shape (hence thedesignation: ulex = mushroom). Germinal matrix hemorrhage, ulegyria, and periventricularleukomalacia are among the causes of perinatal brain injury, which leads toa complex set of neurologic manifestations referred to as cerebral palsy.48A 64-year-old woman comes to her physician for management of her hypertension,which has been treated unsuccessfully for several years. She was recentlyhospitalized for pulmonary edema, and an echocardiogram at that time showeda moderately depressed ejection fraction. She was diagnosed with congestiveheart failure. Her medications include a thiazide diuretic and a calcium channel blocker. She has an allergy to furosemide. Her review of systems is positivefor two-pillow orthopnea and occasional paroxysmal nocturnal dyspnea. On physicalexamination, her blood pressure is 150/80 mm Hg, and her pulse is 80/min andregular. Her lungs are clear, and there are no extra heart sounds. Her extremitiesare without edema. Which of the following is the most appropriate management at thistime?

  A. Add an ACE inhibitor to her regimen

  B. Add an angiotensin II receptor blocking

Page 56: TEST 17

agent to her regimen

  C. Add hydralazine to her regimen

  D. Increase the dose of her calcium channelblocker

  E. Increase the dose of her thiazide diureticExplanation:The correct answer is A. Thispatient has both hypertension and congestive heart failure (CHF). An importantconcept to recognize in the treatment of medical conditions is that certainmedications overlap syndromes and are efficacious in many areas. This "co-treatment"option maximizes each drug in a regimen and often addresses two or more issuessimultaneously. In this case, ACE inhibitors have been shown to be very beneficialin prolonging the survival of CHF patients.Adding an angiotensin II receptor blocking agent to a regimen (choice B) has become an alternative for ACEinhibitor therapy in patients who cannot tolerate these drugs for a varietyof reasons. Although their efficacy in lowering blood pressure appears tobe equivalent to that of ACE inhibitors, no data have shown their survivalbenefit to be similar to ACE inhibitors. Because such a clear mortality benefithas been demonstrated for ACE inhibitors, a compelling reason exists to usethem preferentially in almost all patients with systolic dysfunction.Adding hydralazine to her regimen (choiceC) would certainly help to treat her blood pressure but will donothing in terms of helping her CHF.Increasing the dose of her calcium channel blocker (choiceD) or her thiazide diuretic (choiceE) would most certainly aid in lowering her blood pressure, butneither of these medications has any significant utility for treating eithersystolic or diastolic CHF.49A 51-year-old man is complaining of shortness of breath. The patientdescribes a slowly progressive inability to perform physical activities overthe past 2 years. He reports being able to start an activity, such as golfor tennis, but within a few minutes he feels like "he has lost his breath."There is no associated chest pain, pressure, or discomfort. More recently,he has had significant shortness of breath even at rest. The clinical picture,with the addition of a restrictive pattern on pulmonary function tests andinterstitial infiltrates on the chest x-ray film, suggests the diagnosis ofinterstitial lung disease. Which of the following is the most appropriate next step in management?

Page 57: TEST 17

  A. Follow-up in the clinic in 2 months, asmost interstitial lung diseases are untreatable

  B. Referral to a pulmonologist for furtherevaluation and possible transbronchial biopsy

  C. An empiric course of prednisone for 6 weeks

  D. Immunosuppressive therapy with azathioprine

  E. Referral to a thoracic surgeon for furtherevaluation and possible open lung biopsyExplanation:The correct answer is B. Apulmonologist may be able to help confirm your diagnosis and is best equippedto recommend confirmatory tests, such as transbronchial biopsy, and help directtreatment options.Simply arranging for follow up (choiceA) without confirming the diagnosis is inadequate. It is true thatmany of the interstitial lung diseases have disappointing treatment responses,but there are important new advances all the time, and a tissue diagnosismay be indicated. Overall, patients will be helped by referral to a specialistat this point.Prednisone (choice C) andazathioprine (choice D) are immunosuppressiveagents that are sometimes used to treat the active inflammation seen in someinterstitial lung processes. Because of their severe, systemic side effects,however, they should generally be used in coordination with a lung specialist.Open lung biopsy performed by a thoracic surgeon (choiceE) may ultimately help confirm the diagnosis, but referral to apulmonologist is more appropriate initially. Pulmonologists are more likely to play a role in the confirmatory evaluation and help direct treatment. Theymay then decide to perform a biopsy via bronchoscopy or consult a thoracicsurgeon if indicated.50A 12-month-old infant is brought to the emergency department with biliousvomiting and abdominal distention for 10 hours. His mother states that theinfant has been constipated since birth and failed to pass meconium duringthe first 48 hours of life. When specifically asked, she says that he oftenhas brown, speckled vomitus. On examination, he is very irritable. His length

Page 58: TEST 17

and weight are both below the 5th percentile according to his age. His abdomenis moderately distended. After a digital rectal examination, a fair amountof stool ejects out from the anus. Which of the following is the most likely diagnosis?

  A. Duodenal atresia

  B. Intussusception

  C. Hirschsprung disease

  D. Malrotation

  E. Pyloric stenosisExplanation:The correct answer is C. Thisinfant has Hirschsprung disease, or congenital aganglionic bowel disease.It is five times more common in boys than in girls. It results from congenitalabsence of ganglion cells in either part of or the entire wall of the colon,resulting in a state of chronic contraction. In most cases, the aganglionicsegment is limited to the rectosigmoid colon. In very rare cases, part ofor the entire small bowel can be aganglionic as well. Bilious or feculentvomiting, abdominal distention, and constipation are the classic clinicalsigns. There might also be a history of failure to pass meconium in the first48 hours of life. If only a short segment of the colon is involved, Hirschsprungdisease might not be evident until in childhood or adolescence. Megacolonproximal to the aganglionic segment might be visible on barium enema. Thediagnosis is confirmed with the demonstration of an aganglionic segment ofthe bowel on punch biopsy.Duodenal atresia (choice A) usuallypresents with vomiting. A "double bubble" sign is seen on abdominal radiography.Thirty to forty percent of cases are associated with Down syndrome.Intussusception (choice B) iscertainly in the differential diagnosis of vomiting and abdominal distention.In this case, however, the history of failure to pass meconium in the newbornperiod and failure to thrive is much more suggestive of Hirschsprung disease.Intussusception occurs when one segment of the bowel telescopes into anothersegment just distal to it. The most common site of intussusception is theileocolic junction.Malrotation (choice D) isusually caused by the presence of a volvulus, which presents with sudden onsetof bilious vomiting, abdominal distention, rectal hemorrhage, peritonitis,

Page 59: TEST 17

and shock. It is a surgical emergency.Pyloric stenosis (choice E) usuallypresents with projectile vomiting in the first 2 or 3 weeks of life. On examination,an olive-shaped mass is usually palpable in the epigastric area. It is causedby hypertrophy and hyperplasia of the antrum of the stomach, resulting inobstruction. It occurs in 1 of 150 boys and in 1 of 750 girls.